Vous êtes sur la page 1sur 53

Cataract/Anterior Segment

Question 1 of 130
In determining the appropriateness of proceeding with cataract surgery, a visual functioning index (the VF-14) has been
used to assess the functional status in patients with cataracts. Which of the following statements does not accurately
describe the VF-14?

A. The VF-14 has been shown to be both a valid and internally consistent measure of impairment in patients
with cataracts.

B. The VF-14 was designed to measure functional impairment due to cataracts.

C. Examples of some of the functional activities on the VF-14 include small print, doing fine handwork, and
daytime and nighttime driving.

D. The VF-14 addresses a patient's ability to perform 20 vision-dependent activities.


Please select an answer
Feedback: VF-14 is a brief, 14-question inventory that attempts to measure a patient's real-life visual difficulties by
reviewing day-to-day activities such as driving, reading small print, and doing fine handwork. Clinical testing of the VF-14
questionnaire has shown it to be a reliable and valid measure of practical visual difficulties caused by cataract. It is a highly
reproducible test and is being used more and more in routine clinical practice in addition to research studies. A number of
commercially available clinical databases include variations of the VF-14 or the VF-14 in its entirety.

Question 2 of 130
Which of the following statements does not accurately describe intraocular lens power calculations?

A. A11 theoretical formulas use a variable called the A-constant, anterior chamber depth (ACD) constant, or
surgeon factor.

B. When emmetropia is not the desired postoperative refraction, linear regression formulas are the most
accurate.

C. Formulas such as the SRK/T, Holladay, and Hoffer Q are more accurate than earlier formulas, especially for
eyes with short or long axial lengths.

D. Formulas such as the SRK/T (Sanders-Retzlaff-Kraff theoretic), Holladay, and Hoffer Q are based on clinical
and theoretical optics.
Please select an answer
Feedback: The early IOL power calculation formulas (eg, SRK II) were mainly linear regression formulas derived from
clinical data. More current formulas are based on clinical and theoretical optics and include the Holladay, Hoffer Q, and
Sanders-Retzlaff-Kraff theoretic (SRK/T). These formulas are typically available preprogrammed in newer ophthalmic A-
scan units or with software for most personal computers. They are more accurate than the earlier formulas (eg, SRK I or
SRK II), especially for eyes with short or long axial lengths or when emmetropia is not the desired postoperative refraction.
Theoretical formulas all use a variable called the A-constant, anterior chamber depth (ACD) constant, or surgeon factor.
These variables all relate to the position of the IOL within the pseudophakic eye. With the advent of continuous tear
capsulorhexis and more predictable fixation of the IOL within the capsular bag, it is possible for individual surgeons to refine
their own A-constant and thereby improve the accuracy of their IOL power calculations. The individual surgeon A-constant
can be calculated by reviewing approximately 20 cases from a particular surgeon who is using the same IOL.
Question 3 of 130
A 68-year-old patient previously underwent cataract extraction with insertion of a closed-loop, semiflexible anterior chamber
lens (for example, Leiske, Azar, or Dubroff designs). The patient is asymptomatic, but visual acuity has decreased from
20/20 to 20/30 over the last year. There is a slight irregularity in the shape of the pupil, with a small peak at the 12 o'clock
position. In addition, microcystic edema of the cornea is noted at the superior limbus, extending approximately 2 mm into the
cornea. Which of the following examination techniques or procedures is least likely to be helpful in evaluating these
findings?

A. Corneal pachymetry

B. Gonioscopy

C. Specular microscopy

D. Corneal topography
Please select an answer
Feedback: Gradual endothelial cell loss with eventual corneal endothelial decompensation and development of bullous
keratopathy frequently occurs in patients who have had a closed-loop, semiflexible anterior chamber lens implanted. Serial
cell counts, with evaluation of polymegethism and pleomorphism (specular microscopy) performed at 6- to 8-month intervals,
are important in helping to provide appropriate care to these patients. Corneal pachymetry is also helpful in determining the
health of the endothelial cell monolayer. Pachymetry measurements greater than 600 m clearly denote a compromised
cornea even though the cornea may appear clear clinically. Progressive cell loss and increased corneal thickness greater
than 600 m, with clinical evidence of microcystic edema, are strong reasons for removing the offending lenses. Gonioscopy
is important in determining the position of the intraocular lens haptic and the status of the vitreous face. In this particular
case, the peaked pupil at the 12 o'clock position and the accompanying superior microcystic corneal edema are probably
due to the superior haptic touching the peripheral cornea and to iris capture due to IOL malposition. Corneal topography is of
minimal help in determining the cause of the corneal edema seen clinically in this patient.

Question 4 of 130
Uneventful nucleus removal by phacoemulsification has been accomplished. During the removal of cortical material by
irrigation/aspiration, anterior movement of the entire posterior capsule and loss of posterior chamber volume are noted, and
the anterior chamber becomes shallow but does not collapse. This situation persists, even with the foot pedal in position 1
(infusion only), and no other obvious abnormality is noted. What is the most likely cause of this phenomenon?

A. Empty infusion bottle

B. Choroidalhemorrhage

C. Misdirection syndrome

D. Maneuver by patient
Please select an answer
Feedback: A persistent, forward shift of the posterior capsule during irrigation/aspiration is usually caused by flow of
infusion fluid through either an area of intact zonules or an area of zonular dehiscence. Fluid accumulates between the
anterior hyaloid and posterior capsule, displacing the capsule forward and causing the anterior chamber to become shallow.
Microscopic lens particles can be seen behind tlie posterior capsule in such instances, having been carried through the
zonule by flow of fluid. Although a choroidal hemorrhage is possible, it is extremely unlikely and can be ruled out by
performing indirect ophthalmoscopy. Choroidal hemorrhage is a devastating complication. If there is a suspicion of choroidal
hemorrhage (shallowing of anterior chamber, pain, loss of red reflex, increased intraocular pressure), the eye should be
emergently closed. A Valsalva maneuver or coughing could briefly cause the anterior chamber to become shallow, but the
condition would not persist. An empty infusion bottle could indeed cause the anterior chamber to become shallow, or more
likely, to collapse completely. However, the globe will almost always be visibly soft (ie, corneal striae and/or corneal
concavity become obvious).
Question 5 of 130
In an effort to prevent postoperative endophthalmitis following cataract surgery, which of the following is least efficacious?

A. Instillation of 5% topical povidone-iodine during presurgical preparation of the eye

B. Treatment of blepharitis with hot compresses, lid hygiene, and antibiotic one week before cataract surgery

C. Addition of gentamicin to the intraocular irrigating solution during cataract surgery

D. Keeping the eyelashes out of the field using a plastic adhesive drape
Please select an answer
Feedback: The organisms responsible for 80% to 90% of postoperative endophthalmitis are Gram-positive organisms
commonly found on the eyelids. Molecular studies have confirmed that the vast majority of intraocular infections are caused
by organisms that originate on the patient's own lids. Patients with significant blepharitis or meibomianitis should be treated
aggressively prior to surgery, as meibomian glands may be colonized by bacteria. Patients with acne rosacea or seborrheic
dermatitis should likewise be treated with systemic doxycycline preoperatively to minimize the potential for bacteria in the
operative field. The use of aminoglycoside antibiotics as prophylaxis in intraoperative irrigating solutions poses significant
risk of toxicity to the retina if dilutional errors are made in preparation. Furthermore, aminoglycosides are largely ineffective
against Staphylococcus epidermidis and Streptococcus pneumoniae, which are common causes of postoperative
endophthalmitis. Isolation of the meibomian gland orifices and lashes is bet accomplished with the use of an adhesive
plastic drape. In this way, exposure of the eye to lash and lid bacteria is minimized.

Question 6 of 130
A patient who had radial keratotomy several years ago is scheduled for cataract extraction and intraocular lens
(IOL)implantation. Which of the following statements is most accurate?

A. If standard keratometry is used in the determination of IOL power before cataract extraction, hyperopia
frequently results.

B. If standard keratometry is used in the determination of power before cataract extraction, myopia is likely to
result postoperatively.

C. If standard keratometry is used in the determination of IOL power before cataract extraction, irregular
astigmatism frequently results postoperatively.

D. To accurately determine the IOL power, the axial length must be adjusted (decreased by 0.25 mm) from the
measured length.
Please select an answer
Feedback: The standard for determining IOL power is based on keratometry values and axial length of eye. In patients who
have had radial keratotomy (RK), standard keratometry typically overestimates corneal power because it reads an annular
zone of the cornea near midperipheral knee; this zone is steeper than more central corneal regions. Use of the keratometric
value in IOL calculation can result in the selection of an insufficient power, producing postoperative hyperopia.
Computerized videokeratography (CVK) values at the 1-mm and 2-mm zones would be helpful to this patient. The
keratometric values derived from CVK in IOL calculations are more accurate than those obtained by routine methods.
Irregular astigmatism and myopia are no more likely to occur after cataract extraction following refractive surgery than after
cataract extraction not following refractive surgery. No change axial length is necessary for the calculation of IOL power
following RK.
Question 7 of 130
A 76-year-old patient with a dense nuclear sclerotic cataract is brought to the operating room for phacoemulsification with
IOL insertion. A continuous curvilinear capsulotomy is performed. Two perpendicular grooves are sculpted into the lens
nucleus. The lens nucleus is fractured into four quadrants with the ultrasound tip and spatula. Two nuclear quadrants are
emulsified uneventfully. However, while engaging the third quadrant, a rent in the posterior capsule is noted about the same
time as the remaining nuclear quadrant disappears into the vitreous cavity. The anterior capsular rim remains intact, and
vitreous appears at the wound as the phacoemulsification needle is removed. You are able to emulsify the nuclear piece
engaged on the phacoemulsification tip. Assuming that you are not a retinal specialist, and that one is not immediately
available, what is the recommended course of action?

A. Perform a limited anterior vitrectomy to remove any vitreous from the anterior segment. Remove residual
cortical material with irrigation/aspiration and close the eye.

B. Immediately close the eye and refer the patient to a vitreoretinal specialist.

C. Perform a limited anterior vitrectomy. Remove any residual cortical material. Reintroduce the
phacoemulsification needle into the midvitreous and attempt to engage the remaining quadrant of nuclear material
and emulsify it behind the iris plane. If successful, proceed with placement of a posterior chamber lens within the
ciliary sulcus.

D. Perform a mechanical anterior vitrectomy and remove residual cortical material using irrigation/aspiration.
Insert a posterior chamber lens within the ciliary sulcus. Close the wound and refer the patient to a vitreoretinal
specialist.
Please select an answer
Feedback: Recommended treatment for retained nuclei after cataract surgery includes referral to a vitreoretinal specialist so
that vitrectomy may be performed if the surgeon deems it necessary. Introduction of the phacoemulsification needle into the
vitreous is contraindicated, as the combination of poor vitreous cutting combined with vitreous traction can lead to large
retinal tears. Mechanical anterior vitrectomy should be performed to allow efficient removal of retained cortical material and
vitreous from within the anterior segment in an effort to further minimize postoperative inflammation. Insertion of an
intraocular lens at the time of cataract surgery when dislocated lens fragments are present is not contraindicated provided
the procedure can be performed safely. In this case adequate support is available because the anterior capsular rim is still
intact.

Question 8 of 130
Which of the following statements is least accurate concerning lens anatomy?

The anterior lens capsule is thicker than the posterior lens capsule.

The lens capsule is a basement membrane.

With increasing age, the equatorial zonules become predominant and the posterior zonules are lost.

Zonules insert more centrally on the anterior lens capsule than they do on the posterior lens capsule.
Please select an answer
Feedback: Equatorial zonules are lost with increasing age, and the anterior and posterior zonules predominate. Zonules do
insert more centrally on the anterior lens capsule than they do on the posterior lens capsule. This is important because
occasionally anteriorly located zonules may interfere with the performance of a larger-diameter capsulorhexis. The lens
capsule is a true basement membrane. The anterior lens capsule is thicker than the posterior lens capsule, with the thinnest
area of the capsule in the central posterior capsular region.
Question 9 of 130
Three weeks after uncomplicated phacoemulsification and insertion of a 5.5-mm all-polymethylmethacrylate (PMMA)
posterior chamber lens in the capsular bag, a patient's refraction is +1.50 D in an eye that was planned to be plano
postoperatively. Keratometry shows no astigmatism. Which of the following is most likely to have contributed to this result?

Indenting of the cornea during A-scan measurement

Postoperative posterior vitreous detachment

Too little applanation pressure during A-scan measurement

Nuclear sclerosis-induced myopia


Please select an answer
Feedback: Poor applanation technique during A-scan measurement can result in incorrect measurement of the eye's axial
length. Too much pressure may yield a falsely shallow anterior chamber depth, with an axial length that is inaccurately short.
Too little applanation pressure may result in a fluid bridge between the tip of the probe and the cornea, which results in a
falsely long anterior chamber depth as well as a falsely elongated total axial length. If an axial length measurement is falsely
elongated due to poor applanation technique, the surgeon may implant an intraocular lens with too little power for the eye,
resulting in a hyperopic postoperative correction. Lens-induced myopia should have no effect on the true axial length of the
eye. Likewise, a posterior vitreous detachment should not affect the postoperative refraction.

Question 10 of 130
A 50-year-old insulin-dependent diabetic patient is about to undergo cataract-implant surgery. The patient previously
underwent argon laser retinal photocoagulation for diabetic retinopathy, and the retinal subspecialist has requested that the
upcoming procedure be performed in such a way as to maximize a surgeon's ability to examine and treat the retina in the
future, as well as to facilitate performance of pars plana vitreous surgery or other procedures that might be needed. Which of
the following techniques or characteristics is least likely to create future visualization problems should vitreous surgery be
necessary?

Acrylic IOL

Small-diameter or ovoid IOL optic

Small capsulorhexis

Silicone IOL
Please select an answer
Feedback: A small capsulorhexis could create future visualization problems, although this may be partially alleviated by
Nd:YAG laser anterior capsulotomy. Small-diameter or ovoid IOLs can also contribute to the difficulty a surgeon may have
visualizing the peripheral retina. Silicone IOLs, because of their hydrophobic nature, may develop water droplets on their
posterior surface during the air-fluid exchange procedures that are commonly performed during surgery. This problem can
occur only if the posterior capsule has been opened; IOL removal may be necessary to create adequate visualization for a
vitreoretinal procedure. This problem does not appear to occur when acrylic or polymethylmethacrylate (PMMA) IOLs are
implanted.
Question 11 of 130
Which of the following procedures is inappropriate in managing a dropped nucleus intraoperatively?

Injecting intracameral triamcinolone to identify vitreous in the anterior chamber for anterior vitrectomy

Continue to phacoemulsify the lens in the anterior vitreous to minimize amount of lens falling back

Injecting viscoelastic device (OVD) posterior to the nucleus to lift into anterior chamber, if possible

Placing sulcus IOL and closing the wound with referral to a vitreoretinal specialist for removal of retained lens fragment
Please select an answer
Feedback: Continuing to phaco in the vitreous is inappropriate in managing a dropped nucleus intraoperatively because it
may cause traction on the retina causing retinal tears or breaks and add additional risk for retinal detachment.

Question 12 of 130
A patient undergoes a combined cataract and filtering procedure. On the first postoperative day the patient has an
intraocular pressure (IOP) of 43 with a chamber that is flat peripherally and very shallow centrally with no lens-corneal touch.
What is the most likely diagnosis?

Angle closure

Aqueous Misdirection

Choroidal effusion

Wound leak
Please select an answer
Feedback: Markedly elevated IOP and flat chamber.

Question 13 of 130
What is one of the factors that minimize surgically induced astigmatism when performing cataract surgery?

Size of the wound

Bimannual versus Coaxial Irrigation & Aspiration technique

Scleral Tunnel

Chopping versus Divide & Conquer technique


Please select an answer
Feedback: Larger wounds induce more astigmatism.
Question 14 of 130
Which of the following best describes the "divide and conquer" technique of nuclear disassembly?

Is best suited for a soft nucleus relative to chopping

Requires blind maneuvers

Requires prior hydrodelineation

Requires the simultaneous use of 2 instruments


Please select an answer
Feedback: Chopping requires denser nuclei to cleave.

Question 15 of 130
A patient with pseudoexfoliation syndrome is at increased risk for which of the following complications?

Zonular damage during cataract surgery

Corneal edema following cataract surgery

Myopic refractive surprise after cataract surgery

Cystoid macular edema following cataract surgery


Please select an answer
Feedback: Patients with pseudoexfoliation syndrome have increased zonular weakness and have increased risk for surgical
complications such as zonular loss/damage. For uncomplicated cataract surgery, they are not at increased risk for macular
edema, corneal edema, and post-op refractive surprises.

Question 16 of 130
What would be an unlikely cause of post-operative flat anterior chamber after intraocular surgery?

Choroidal Effusion

Wound leak

Endophthalmitis

Aqueous misdirection
Please select an answer
Feedback: Endophthalmitis will often present with a hypopyon. Aqueous misdirection and choroidal effusion will cause
flattened anterior chamber depth. Wound leaks can lead to flat chamber since aqueous leaks out faster than being
produced.
Question 17 of 130
A Longterm sequelae of a postoperative shallow or flat anterior chamber may be which of the following?

A. Glaucomatous cupping

B. Vitreous Hemorrhage

C. Capsular Block Syndrome

D. Corneal Decompensation
Please select an answer
Feedback: Glaucomatous cupping is not a sequelae of flat anterior chamber.

Question 18 of 130
During cataract surgery, which of the following is likely to put patients at higher risk of developing postoperative corneal
edema?

Age related macular degeneration

Use of topical steroids

Posterior chamber phacoemulsification

Fuchs' dystrophy
Please select an answer
Feedback: Fuch's patients are predisposed to post operative corneal edema due to pre-operative endothelial cell loss.
Ultrasound power delivered away from the corneal endothelium is protective. Age related macular degeneration not a risk
factor for corneal edema. Use of topical steroid post-operatively is to help reduce corneal edema

Question 19 of 130
Topical anesthesia may be contraindicated for which of the following pre-operative states?

Monocular patients

Nystagmus

Dementia

Long axial length eyes


Please select an answer
Feedback: Continuous eye movement may not allow for safe operating.
Question 20 of 130
Surgical repositioning of a posterior chamber intraocular lens (PCIOL) in the presence of pupillary capture often involves
which of the following?

Breaking iris adhesions

Replacing the lens with an angle-fixated anterior chamber IOL of a different power

Pretreatment with two YAG iridotomies, at least 90 degrees apart

At least 3 corneal and 1 pars plana incisions to allow the surgeon multiple approach to the PCIOL
Please select an answer
Feedback: Need to break iris adhesions to PCIOL creating pupillary capture. No pars plana incision needed since not
planning to perform pars plana vitrectomy.

Question 21 of 130
When performing an anterior vitrectomy during complicated cataract surgery, which cutting speed setting is the most
appropriate?

Any setting is equally appropriate.

High

Medium

Low
Please select an answer
Feedback: High speed cutting settings are the most appropriate when performing an anterior vitrectomy. High cutting rates
relative to the aspiration flow rate reduces the risk of vitreoretinal traction and retinal tears during vitrectomy.

Question 22 of 130
A 65-year-old woman has a visually significant cataract and moderately high (-7.00 D) myopia in each eye. For the past 20
years, she has successfully worn contact lenses to give her monovision (-3.00 D in the left eye) and does not like to wear
eyeglasses. What is the best plan for postoperative correction and intraocular lens (IOL) choice for this patient?

Emmetropia in the right one eye and about -3.00 D in the left eye

Emmetropia for both eyes

Aim for -7.00 D in both eyes so she can continue the same-strength contact lenses

-3.00 D for both eyes


Please select an answer
Feedback: For this contact lens-wearing monovision patient, aiming for emmetropia in the distance eye and about -3.00 D
for the near eye is the best option. A long history of adaptation to a particular monovision correction (e.g. emmetropia in one
eye and about -3.00 diopters in the near-vision eye) is a useful predictor of success with pseudophakic monovision.
Question 23 of 130
A patient with pseudoexfoliation and a very large, hard, brunescent nuclear sclerotic cataract has marked phacodonesis.
What is the best surgical approach?

Phacoemulsification

Extracapsular cataract extraction

Phacoemulsification employing an endocapsular ring

Intracapsular cataract extraction


Please select an answer
Feedback: With a large, dense nucleus in an eye with pseudoexfoliation and marked phacodonesis, intracapsular cataract
removal is the best approach. Other choices have a substantial risk of the lens being dropped into the vitreous.

Question 24 of 130
Which one of the following cataract types is at increased risk for posterior capsule rupture during hydrodissection in cataract
surgery?

Anterior polar

Posterior polar

Cortical

Posterior subcapsular
Please select an answer
Feedback: The central posterior capsule may be thinned in the patient with a posterior polar cataract, increasing the risk for
posterior capsule rupture during hydrodissection. The rate of posterior capsular rupture in cases using hydrodissection is
approximately 26% (Osher et al., JCRS 1990). Alternative approaches to cortical dissection include hydrodelineation and
viscodissection.

Question 25 of 130
Early in the sculpting phase of a routine cataract phacoemulsification, the entire remaining nucleus drops posteriorly onto
the retina. What is the best course of action?

Enlarge the operative wound and lift the nucleus with a lens loop.

Close the eye and refer the patient to a vitreoretinal specialist.

Install viscoelastic agent and perform deep vitreal phacoemulsion of the nucleus.

Perform an anterior and middle vitrectomy and remove the nucleus and other cataract debris with the vitrector.
Please select an answer
Feedback: A patient with a dropped nucleus should be referred to a vitreoretinal specialist for further management. An
anterior chamber intraocular lens (IOL) can be placed in the eye by the cataract surgeon during the cataract operation.
Question 26 of 130
A patient with no prior history of glaucoma had recent complicated cataract surgery with an anterior chamber intraocular lens
(IOL) inserted. There are many cortical and nuclear fragments in the vitreous, and the intraocular pressure (IOP) is 40 mm
Hg on timolol, dorzolamide, and brimonidine. What is the most appropriate surgery?

Pars plana vitrectomy

Trabeculectomy with mitomycin C

Anterior chamber washout

Glaucoma implant (tube-shunt)


Please select an answer
Feedback: A pars plana vitrectomy is required to remove large amounts of remaining nuclear & cortical lens remnants.

Question 27 of 130
What is the best management for phacolytic glaucoma?

Glaucoma implant (tube-shunt)

Trabeculectomy

Intensive topical corticosteroid therapy

Cataract extraction
Please select an answer
Feedback: In phacolytic glaucoma, the cataract needs to be removed with or without intraocular lens implant insertion.
Glaucoma surgery is usually not required and may cause hypotony. Topical corticosteroids are of little benefit in phacolytic
glaucoma, but may be employed after the cataract surgery.

Question 28 of 130
A patient presents 4 months after routine, uncomplicated cataract surgery with chronic mild anterior chamber cell and flare
and whitish deposits in the capsular bag. There is no hypopyon and the inflammation is only partially responsive to intensive
topical corticosteroid therapy. What is the most likely causative organism?

S. epidermidis

P. acnes

S. aureus

P. aeruginosa
Please select an answer
Feedback: The clinical scenario described best fits a P. acnes intraocular infection. S. aureus and P. aeruginosa tend to
produce a very acute and virulent endophthalmitis. Treatment requires vitreoretinal expertise and can include intraocular
antibiotics, vitrectomy, and subtotal or total capsulectomy with intraocular lens removal or exchange.
Question 29 of 130
Glaukomflecken most often occur after which one of the following?

Vitrectomy in a phakic eye

Acute primary angle-closure glaucoma

Blunt ocular trauma

Attack of severe uveitis


Please select an answer
Feedback: Glaucomflecken are seen in phakic eyes after episodes of very severe IOP elevation such as after acute primary
angle-closure glaucoma.

Question 30 of 130

Insertion of which one of the following intraocular lens (IOL) types usually requires that a surgical iridectomy be performed?

Multifocal IOL inserted in the capsular bag

Posterior-chamber IOL inserted in the capsular bag

Posterior-chamber IOL inserted in the sulcus

Anterior chamber IOL


Please select an answer
Feedback: In the absence of a pre-existing laser iridotomy or surgical iridectomy, patients receiving an anterior chamber
intraocular lens (IOL) require a surgical iridectomy to prevent the risk of pupillary block glaucoma.

Question 31 of 130
Which of the following best describes how hypotony can occur?

From post operative aqueous misdirection

From retained viscoelastic at the time of cataract surgery

From a cyclodialysis cleft associated with traumatic iridodialysis

From posterior subcapsular cataract


Please select an answer
Feedback: A cyclodialysis cleft will increase uveo-scleral outflow and cause lowering of intraocular pressure. Retained OVD
and Aqueous misdirection will cuase elevated intraocular pressure. Presence of posterior subcapsular cataract should not
affect intraocular pressure.
Question 32 of 130
Which of the following factors may lead to a decision to perform penetrating keratoplasty in combination with cataract
surgery (triple procedure)?

Decreased corneal endothelial polymegathism

Corneal pachymetry measurement > 640 um

Decreased corneal endothelial pleomorphism

Corneal endothelial cell count > 1000 cells/mm2


Please select an answer
Feedback: Corneal thickness is often used as a measure of endothelial cell function in Fuch's Dystrophy patients and
thickness greater than 640 microns may be an indication for corneal transplantation. Endothelial cell count and cell
morphology can also be used to gauge endothelial cell health; the greater the pleomorphism and polymegathism, more likely
to need corneal transplantation.

Question 33 of 130
Which of the following statements describes the optimal size for a continuous curvilinear capsulorrhexis (CCC)?

Optimal size can best be determined by first staining the anterior zonular attachment zone.

Optimal size should be just small enough to fit inside the patient's scotopic pupil size.

Optimal size should be large enough to expose all of the optic when using a mutifocal IOL.

Optimal size should be large enough to allow for cataract removal and small enough to overlap the anterior optic edge
of the intraocular lens (IOL).
Please select an answer
Feedback: Ideally, the CCC should be centered and made as large as possible but still covering the optic edge to prevent
PCO.

Question 34 of 130
In the surgical removal of an intumescent cataract there is an increased risk of which of the following?

Iris prolapse

Choroidal effusion

Corneal wound burn

Capsular radial tear


Please select an answer
Feedback: Intumescent lenses tend to have a bulging anterior capsular surface so that while making the capsulorhexis, the
tension from the expanding lens material can create radial tears, and making the anterior capsule difficult to manage.
Question 35 of 130
Which of the following steps can be taken to reduce the operative risks associated with cataract surgery in an eye that has
had a previous penetrating keratoplasty?

Injection of a large, filtered air bubble into the anterior chamber at the conclusion to give the endothelium extra
protection in the early postoperative period

Treatment of the cornea with mitomycin C to decrease the risk of graft rejection or failure

Performance of a phacoemulsification as deeply as is safe to avoid endothelial trauma and consideration of


extracapsular cataract extraction (ECCE)

Increasing the aspiration flow rate and duty cycle on the phaco machine to decrease the surgery time
Please select an answer
Feedback: The goal is to minimize endothelial cell trauma with using as little phaco power as needed as far away from the
endothelium as possible and using Mitomycin C on the endothelium will cause cell death.

Question 36 of 130
A pianist had cataract surgery and intraocular lens implantation in the capsular bag 2 months ago. Her best-corrected acuity
in the operative eye is 20/15 with +1.25 sph. The fellow eye is phakic, 20/15 with -0.75 sph. She complains bitterly about
blurred vision in the operative eye and does not want to wear glasses or contacts. Which of the following options is the most
appropriate?

implantation of a +3.0 D piggyback IOL

laser thermal keratoplasty (LTK) to permanently correct the hyperopia

hyperopic LASIK to make patient -1.00 D for near

IOL exchange with premium IOL and option for lasik enhancement
Please select an answer
Feedback: Since the patient is less than 3 months pos-op, IOL exchange is still a good option. Discussion is needed to
educate patient on multifocal or pseudo-accomodating IOL to correct for presbyopia with the IOL exchange.

Question 37 of 130
What did the Endophthalmitis Vitrectomy Study (EVS) conclude when a patient's vision at presentation was light perception
only?

The visual outcome was better with anterior chamber paracentesis and intracameral antibiotics.

The visual outcome was better with pars plana vitrectomy and intravitreal antibiotics.

There was no difference in the visual outcome between all treatment groups.

The visual outcome was better with pars plana vitrectomy and no intravitreal antibiotics.
Please select an answer
Feedback: EVS study concluded that intravitreal antibiotics and pars plana vitrectomy for light perception vision or worse to
be the best treatment protocol for visual outcome. For vision better than light perception, pars plana vitrectomy and
intraviteal antibiotic versus vitreous diagnostic tap and intravitreal antibiotics resulted in no difference in visual outcome.
Question 38 of 130
When choosing a foldable intraocular lens (IOL) for implantation during cataract surgery, which of the following is the most
compatible lens for use with silicone oil in the posterior chamber?

Plate style silicone IOL

Hydrophobic acrylic IOL

Three-piece silicone IOL

Polymethylmethacrylate (PMMA) IOL


Please select an answer
Feedback: Silicone IOLs should not be placed in an eye with silicone oil.

Question 39 of 130
The management of a large Descemet's detachment following a cataract procedure may involve which of the following?

Reposition tear with OVD

Filling half the chamber with SF6 gas and positioning the head to help seal

Hourly steroid drops

Observation
Please select an answer
Feedback: A gas bubble either with SF6 gas or air can be used to tamponade the Descemet's flap into position until
adherence is made.

Question 40 of 130

Which of the following statements represents the best treatment strategy for clinically significant diabetic macular edema in a
patient about to undergo cataract surgery?

Pre-existing diabetic macular edema should be treated with focal laser before the cataract is removed.

Pre-existing diabetic macular edema should be treated simultaneous to the cataract surgery with intraoperative
endolaser.

Pre-existing diabetic macular edema should be treated with focal laser after the cataract is removed.

The insulin dose should be decreased to 1/2 of the usual morning amount on the day of cataract surgery, if the patient
has had no food or drink (NPO) after midnight.
Please select an answer
Feedback: Pre-existing diabetic retinopathy should be stabilized before elective cataract removal to minimize risk of
worsening or progressive complication.
Question 41 of 130
Phacoemulsification is performed after the creation of a continuous, circular capsulotomy. At the completion of nucleus and
cortex removal, a radial anterior capsular tear is noted at 6 0'clock, and the posterior capsule appears to be intact. In this
situation, which of the following statements is least accurate?

A single-piece foldable IOL can be placed in the capsular bag.

A multipiece foldable IOL can be placed in the ciliary sulcus.

A multipiece foldable IOL can be placed in the capsular bag.

A single-piece foldable IOL can be placed in the sulcus.


Please select an answer
Feedback: A multipiece foldable IOL can be placed in the capsular bag or sulcus (with sufficient capsular support) since
these lenses do not rotate after insertion. Single-piece foldable I0Ls should not be implanted in the sulcus since they may
migrate and escape out of the ciliary sulcus. These lenses should not be placed into the ciliary sulcus, because
pseudophacodonesis will occur with current designs, and pigment release and/or recurrent iritis may ensue. A multipiece
foldable I0L may be placed into the ciliary sulcus if inserted perpendicular to the radial tear to prevent sun-setting if the
posterior capsule is involved.

Question 42 of 130
A 23-year-old woman requests refractive surgery. She is a -3.00 myope. Keratometry reading in the right eye is 44.00
D/45.00 D x 90 degrees. Keratometry reading in the left eye is 45.00 D/48.000 x 120 degrees with +2 distortion of the
keratoscopic mires OS. What is the most appropriate advice to give her?

Discontinue her contact lenses for 2 to 3 weeks and return for corneal topography

Discontinue her contact lenses indefinitely

Have radial keratotomy (RK) in the right eye and an RK with transverse incision in the left

Increase artificial tears, since they may help the irregular mires
Please select an answer
Feedback: This patient may have distortion of corneal mires in the left eye based on either contact-lens-induced corneal
warpage or early keratoconus. It would be best to have her discontinue the lenses for 2 to 3 weeks and then reevaluate the
keratometry and, if possible, perform corneal topography. Topography that is characteristic of keratoconus may be
diagnostic. Refractive surgery is not indicated in patients with keratoconus because the long-term effects of a corneal
weakening procedure, such as radial keratotomy, or a subtraction procedure, such as excimer photoablation, are unknown.
Artificial tears would not affect corneal irregularity unless there was irregular drying of the surface.
Question 43 of 130
Which of the following statements about extracapsular cataract extraction in a patient with diabetes mellitus is least
accurate?

Overweight women with diabetes have a significantly worse visual outcome than patients with normal body weight and
diabetes.

The cataract procedure of choice in diabetic patients is a small-incision phacoemulsifica tion with a 5.0 mm optic IOL
and sub-5mm capsulorhexis opening to prevent decentration.

Cataract extraction is highly associated with progression of nonproliferative retinopathy in the operated eye.

Patients with preexisting nonproliferative diabetic retinopathy have a worse visual prognosis than those without
retinopathy.
Please select an answer
Feedback: Patients with symmetric nonproliferative retinopathy who underwent extracapsular cataract extraction and
intraocular lens implantation were followed postoperatively to determine the incidence of progression of diabetic retinopathy,
visual acuity, and factors predictive of progression of retinopathy in final visual acuity. Progression of retinopathy, defined as
development of clinically significant macular edema and an increase in intraretinal hemorrhages or hard exudate, was
assessed. Cataract extraction was highly associated with asymmetric progression of nonproliferative retinopathy;
progression occurred only in the operated eye in 7 of 19 patients (37%), but in no patients did progression occur in the fellow
eye alone. Women had a significantly increased risk of progression, and overweight women had a significantly worse visual
outcome than those patients with normal body weight. Because of the increase in diabetic retinopathy following cataract
extraction and intraocular lens implantation, large-optic intraocular lenses should be placed at the time of surgery with large
capsulorhexis openings to allow adequate peripheral visualization for panretinal photocoagulation, which may be necessary
months to years following the procedure.

Question 44 of 130
Which of the following is not a sign of a posterior capsular rupture during phacoemulsification?

An area of the posterior capsule that appears "too clear" compared to adjacent areas

The appearance of vitreous in the anterior chamber

Slight deepening of the anterior chamber

Constriction of the pupil


Please select an answer
Feedback: Deepening of the anterior chamber and vitreous in the anterior chamber are both early signs of a posterior
capsular rupture. Additionally, the equatorial lens capsule may come into view if there is a zonular dehiscence. If an area of
the capsule appears "too clear," one must also be suspicious of an opening in the capsule, with the clear area representing
the open capsule. Constriction of the pupil is not typically associated with posterior capsular rupture.
Question 45 of 130
Expected postoperative symptoms in the first week following radial keratotomy does not include which of the following?

Metamorphopsia

Starbursting

Foreign-body sensation

Visual fluctuation
Please select an answer
Feedback: During the first few weeks following radial keratotomy (RK), some fluctuation of vision is expected. This is the
result of resolving edema around the incision wounds. The edema contributes to increased midperipheral wound gaping; this
is reflected topographically as increased midperiphera1 steepening and concomitant central flattening. The increased
flattening, of course, refractively causes reduced convergence power. As the peri-incisional edema resolves, this artifactual
flattening diminishes. Refractively, this results in reduced effect of the procedure. Fluctuation of vision may persist for many
months due to the normal diurnal fluctuation of corneal edema from a closed-eye to open-eye state. Both starbursting and
halos are seen after RK. They appear to be the result of the (expected) prolonged time of corneal incision wound healing.
Most patients experience these phenomena on the first day postoperatively along with a foreign-body sensation from the
incisions. Usually, they will diminish significantly over a period of several weeks to months. Rarely, they may persist
indefinitely. Metamorphopsia is an uncommon complication of radial keratotomy.

Question 46 of 130
At the conclusion of cataract extraction, an inferior zonular dialysis of approximately 90 degrees is noted. The capsule is
otherwise completely intact without a tear or rent. Which of the following statements concerning intraocular lens insertion in
this situation is most accurate?

A single or multipiece PMMA IOL may be inserted into the ciliary sulcus, and the preferred orientation of the long axis
of the implant is vertical (in the axis of the dialysis).

Any appropriate intraocular lens may be inserted into the capsule, and the preferred orientation of the long axis of the
implant is horizontal (perpendicular to the axis of the dialysis).

A single or multipiece PMMA IOL may be inserted into the ciliary sulcus, and the preferred orientation of the long axis
of the implant is horizontal (perpendicular to the axis of the dialysis).

The patient should receive no intraocular lens, since any lens is likely to dislocate.
Please select an answer
Feedback: In cases of a zonular dialysis that is equal to or less than 180 degrees in extent, many patients will do well with
intraocular lenses placed into the capsule. A longer, stiffer lens with a larger optic might be less prone to decentration or to
visual symptomatology from mild degrees of decentration. Most surgeons recommend that the long axis of the implant be
placed in the axis of the dialysis, ie, the haptics should be placed in the axis of the dialysis; this might prevent or minimize
decentration by resisting contraction of the capsule. Placement of the implant into the ciliary sulcus is also possible. If this
method of fixation is selected, however, the long axis of the implant should be placed perpendicular to the axis of the
dialysis. If the implant is placed so the haptic(s) are in the region of the dialysis, decentration of the implant may occur, and
Question 47 of 130
Which of the following is not a component of one or more of the commercially available viscoelastic materials?

Sodium hyaluronate

Keratan sulfate

Hydroxypropyl methyl cellulose

Chondroitin sulfate
Please select an answer
Feedback: Sodium hyaluronate, a mucopolysaccharide found in connective tissues, possesses a high viscosity and high
molecular weight. It is the exclusive component in Healon, AmviscPlus, Vitrax, and ProVisco Chondroitin sulfate, one of the
mucopolysaccharides of the cornea, possesses medium molecular weight and medium viscosity. Chondroitin sulfate is a
component of Viscoat along with sodium hyaluronate. Hydroxypropyl methyl cellulose is a cellulose polymer not naturally
occurring in animals. It is a structural substance in plant fibers and possesses a low molecular weight and low viscosity. It is
the major component of OcuCoat. Keratan sulfate, a mucopolysaccharide found in the cornea, is not a component of any
currently available viscoelastic material.

Question 48 of 130
Lid ecchymosis, subconjunctival hemorrhage, and proptosis appear immediately after retrobulbar anesthetic injection prior to
cataract surgery. The globe is rock hard, immobile, and cannot be displaced posteriorly. Immediate management should
include which of the following?

Vitreous aspiration to reduce posterior pressure

Lateral canthotomy and, if necessary, paracentesis to reduce intraocular pressure

Intermittent massage of the globe to reduce pressure prior to proceeding with surgery

Topical ocular hypotensives for pressure and lateral tarsorrhaphy to reduce proptosis and exposure keratitis
Please select an answer
Feedback: Retrobulbar hemorrhage can cause severe elevation in the intraocular pressure due to posterior pressure from
the orbit. Steps should be taken to relieve orbital tension with lateral canthotomy to give more space for the hemorrhage to
displace.

Question 49 of 130
During phacoemulsification a capsular break is noted with vitreous in the anterior chamber. Nucleus removal is incomplete.
Which of the following steps should not be taken?

Consider conversion to an extracapsular procedure with utilization of a lens loop to remove the nuclear fragment

Remove all vitreous from the anterior chamber using an automated vitrector and low inflow

Inject viscoelastic beneath the nuclear remnant to keep it elevated

Continue gentle phacoemulsification, since the phacoemulsification instrument efficiently cuts vitreous
Please select an answer
Feedback: When a capsular break is noted during phacoemulsification and nucleus removal is incomplete, strategies must
be undertaken to remove the nuclear fragment(s) and avoid the potential for a "dropped" nucleus. Visualization of the limits
of the break, or dehiscence, must be made. If the break is enlarging, it is best to stop and convert to an extracapsular
procedure. The wound should be enlarged and viscoelastic should be injected beneath the nuclear remnant to keep it
elevated. The nuclear fragment can be "tire-ironed" up with a cyclodialysis spatula, and a lens loop can be inserted to
remove the nuclear fragment(s). Expression-type techniques should be avoided, since these will increase vitreous pressure.
All vitreous can then be removed from the anterior chamber using low-inflow vitrectomy to minimize extension of the rent. If
the capsular break is not enlarging, the surgeon should check for external pressure on the lid and globe by the speculum.
The bottle should be lowered to minimize inflow and, hence, extension of the break. The anterior chamber should be swept
with a cyclodialysis spatula to check for the presence of vitreous. Inefficient aspiration either in the irrigation/aspiration or
phacoemulsification mode indicates that vitreous is present in the anterior chamber. Careful phacoemulsification can
proceed if the extent of the rent is visualized, and a second instrument can be used to prevent nuclear fragments from
migrating through the capsular opening. Some surgeons advocate the use of a Sheets glide (a small piece of plastic) that is
slid under the nucleus and over the capsular opening, thus protecting against loss of nuclear material through the rent as
phacoemulsification proceeds. Following removal of the nucleus, retained cortex can be removed by pulling the cortical
material toward the break rather than away from the break, which can extend the rent. During cortical removal, a second
irrigating port can be utilized to minimize inflow. A "dry technique" cortical removal can also be used, in which the anterior
chamber is filled with viscoelastic and a manual aspirating cannula is used to remove residual cortical material. Importantly,
the phacoemulsification tip is not a vitreous cutter and should never be used to remove vitreous.

Question 50 of 130

Which of the following statements regarding preoperative testing in a patient with biomicroscopic evidence of cataract is
most accurate?

Patients at risk for corneal decompensation from surgery are often difficult to identify through history and clinical
examination. Specular microscopy should be routinely performed in patients anticipating cataract extraction by
phacoemulsification.

In a patient with good Snellen acuity and complaints of glare, glare testing should be performed as part of the
preoperative evaluation.

Contrast sensitivity will help differentiate between visual loss due to the cataract and visual loss from a macular
problem.

In eyes with opaque media and vision of 20/200 or worse, potential visual acuity testing with interferometry provides an
accurate estimate of visual outcome and should be performed.
Please select an answer
Feedback: It is reasonable to perform a glare test as part of the evaluation of patients who complain of glare, have
symptoms potentially attributable to glare, or who have a cataract (particularly a posterior subcapsular cataract) and good
Snellen acuity. However, glare testing should not be required by utilization review or quality assessment organizations (eg,
peer review organizations) as objective documentation of visual disability sufficient to justify the potential benefit of surgery.
Glare testing is not useful in patients with cataract who do not complain of symptoms potentially attributable to glare
regardless of their visual acuity. Contrast sensitivity testing does not differentiate between visual loss due to cataract and
visual loss from other causes. Also, in eyes with opaque media and vision of 20/200 or worse, none of the tests of potential
vision provide an accurate estimate of visual outcome after uncomplicated cataract surgery. Electrophysiologic tests may,
however, be useful in determining whether light signals are being received by the retina or brain. Evidence is also lacking in
the published literature that potential vision measurement increases the accuracy of the predicted outcome beyond that
based on history and ocular examination alone. There is currently no evidence in the published literature and no compelling
rationale to support routine use of specular microscopy in patients for whom cataract surgery is being considered. Most
patients at risk of corneal decompensation can be identified through history and clinical examination.
Question 51 of 130
Which of the following is least likely to be a cause of a shallow or flat anterior chamber following cataract extraction?

Ciliary-block glaucoma

Postoperative wound leak

Pupillary-block glaucoma

Viscoelastic glaucoma
Please select an answer
Feedback: A shallow or flat anterior in the immediate postoperative period may be associated with a low or high intraocular
pressure (IOP). A low IOP suggests a wound leak, retinal detachment, or cyclodialysis cleft. A high IOP can be secondary to
pupillary-block or ciliary-block glaucoma. Ciliary-block glaucoma is a rare surgical complication in which the anterior
chamber either fails to form or diminishes secondary to aqueous misdirection. A flat or shallow anterior chamber in the
presence of a patent iridectomy with elevated pressure is typical of the condition. The forward flow of aqueous is blocked by
the ciliary body and diverted posteriorly into the vitreous. Pupillary-block glaucoma results in a shallow peripheral anterior
chamber due to impedance of aqueous flow from the posterior to the anterior chamber. The aqueous "collects" or becomes
trapped between the anterior surface of the posterior chamber IOL and the posterior surface face of the iris. As pressure in
the posterior chamber increases, the peripheral iris is pushed forward to cover the trabecular meshwork and close the angle.
The forward movement of the iris also shallows the anterior chamber. Retained viscoelastic material causes blockage of the
trabecular meshwork and a secondary open-angle glaucoma and does not affect anterior chamber depth.

Question 52 of 130
A patient about to undergo cataract extraction and lens implantation has had a previous pars plana vitrectomy with
intravitreal silicone oil. Standard keratometry and axial length measurements are taken, and the intraocular lens (IOL) power
is computed using one of the linear regression formulas (eg, SRK II) or theoretical formulas (eg, SRK/T or Holladay). Which
of the following best describes the difference between the predicted and postoperative refraction when a biconvex IOL is
placed?

The high viscosity of silicone oil induces a greater than predicted with-the-rule astigmatism.

The postoperative refraction is more hyperopic than would have been predicated by the formulas.

The postoperative refraction is more myopic than would have been predicted by the formulas.

There is no difference between the predicted and postoperative refraction.


Please select an answer
Feedback: Studies have shown that biconvex or plano-convex intraocular lens power calculations using either regression or
formulas yield an average of 4.0 D of hypermetropic shift in patients in whom silicone oil replaces the vitreous. The silicone
shifts the final refraction in a hyperopic direction for two reasons: (1) the actual power of the implant is reduced when there is
any power on the back surface (biconvex or plano-convex) and (2) light traveling through the higher index of refraction
silicone will be slowed down, thus lengthening the focal point. This power shift can be explained by the higher refractive
index of silicone oil (1.405) compared with that of vitreous (1.336). The higher refractive index changes the effective power
of the implanted intraocular lens. The optical power of posterior surface of a biconvex IOL is directly proportional to the
difference between the refractive index of the lens material (eg, polymethylmethacrylate, or PJMMA, with an index of
refraction of 1.491) and that of the material in the vitreous space. For vitreous, this difference is 0.155 (1.491 - 1.336); for
silicone oil, it is smaller: 0.086 (1.491 - 1.405). Thus the optical power of the posterior surface of a biconvex or plano-convex
IOL is effectively reduced when the vitreous cavity is replaced with silicone oil and the patient's refraction becomes
hyperopic relative to the expected refraction. The hyperopic shift can be minimized by using a convex-plano because the
posterior surface is plano. This configuration of the IOL eliminates the reduction in power of the posterior surface. The
presence of silicone in the vitreous cavity is one of the only indications for specifically implanting a convex-plano IOL. Use of
a convex-plano IOL also minimizes the change in refraction if the silicone is removed. The Holladay IOL Consultant
performs all these calculations automatically for the surgeon, who simply has to check the box labeled Silicone in the
Vitreous. The program uses two factors just described to arrive at the appropriate power for any configuration and material
IOL even though the convex-plano lens is recommended.The IOL Master also calculates appropriate IOL powers for
silicone-filled eyes.

Question 53 of 130
A 41-year-old man is evaluated for a unilateral cataract. Examination reveals 20/200 visual acuity with a dense nuclear
sclerotic cataract. Examination of the opposite eye shows it to be entirely normal. The patient denies any trauma except for
multiple occasions when foreign bodies were removed from the cornea after he was injured at his job as an auto mechanic.
Further examination shows a blue iris in the noncataractous eye and a greenish iris in the cataractous eye. In addition, three
or four reddish brown pigment clumps are noted just below the lens capsule in the affected eye. At this point, what is the
most appropriate diagnostic or therapeutic option?

Test for serum copper level

Perform a CT scan

Perform glare testing

Proceed with phacoemulsification and intraocular lens


Please select an answer
Feedback: Iron in the eye, either from blood or an iron foreign body can cause a mild inflammation that simulates other
types of iritis. An inflamed blue iris assumes a greenish hue to siderosis, but the green iris of siderosis bulbi develops after
weeks or years and is typically a darker green. A CT scan to rule out retained iron should be performed if there is suspicion
of an intraocular iron foreign body. Rust spots beneath the anterior capsule of the lens is a typical feature of siderosis. In its
early stages, siderosis produces electroretinogram (ERG) responses that are larger than normal, with low-voltage responses
produced later. A diminution of the b-wave in both light- and dark-adapted conditions can be seen. Ocular chalcosis is a
copper deposition in Descemet's membrane, in the lens capsule (sunflower cataract), and in other intraocular basement
membranes. A foreign body with a high copper content (greater than 90%) causes an acute purulent reaction; lesser
concentrations can be relatively inert. Determining serum copper levels is helpful in identifying Wilson's disease, which is
characterized by copper deposition in Descemet's membrane (ie, the Kayser-Fleischer ring). Although this patient may
eventually undergo cataract extraction for his dense cataract, the identification of iron in the eye portends a poor prognosis.
This is due to the toxicity of iron to retinal cells as a result of its interference with the cytochrome oxidase system. Glare
testing would not be helpful in the diagnosis of this condition.

Question 54 of 130
For any given diopter power, the higher the refractive index, the thinner the intraocular lens optic. Which of the following
currently available intraocular lens optic materials possesses the highest refractive index?

Polymethylmethacrylate (PMMA)

Acrylic

Silicone

Hydrogel
Please select an answer
Feedback: The refractive index of PMMA is approximately 1.47. Hydrogel materials and silicone materials are slightly lower
in refractive index, although new polymers are under development that approach, and in some cases, surpass the refractive
index of PMMA. The refractive index of one acrylic material currently available is approximately 1.55. Hence, thinner lenses
can be made, facilitating insertion through a smaller incision. A 6-mm-diameter optic that is biconvex in configuration and 21
D in power differs in thickness from 0.75 mm with an acrylic material to 0.93 mm in a PMMA material to 1.27 mm with
hydroxyethylmethacrylate (HEMA) to 1.82mm for first-generation silicone.
Question 55 of 130
A two-plane (stepped) clear-corneal incision is performed temporally. An initial groove 3.2 mm in width is created and carried
to a depth of approximately one-half corneal thickness. A 2.5-mm keratome is then used to complete the entry into the
anterior chamber. The site of entry into the anterior chamber is approximately 2.5 mm from the original perpendicular
groove, accomplishing a tunnel 2.5-mm long. Following capsulorhexis and hydrodissection, the phacoemulsification needle
is introduced into the anterior chamber and phacoemulsification is initiated. During phacoemulsification, multiple striae are
noted emanating from the site of the wound. Which one of the following statements best describes this situation?

By raising the bottle, the corneal striae will disappear.

By enlarging the internal and external incision, the striae will disappear.

By decreasing the aspiration rate, the striae will disappear.

By decreasing the vacuum, the striae will disappear.


Please select an answer
Feedback: A standard phacoemulsification needle with the attached sleeve is approximately 2.3 mm in diameter. Placing
this needle through a 2.5-mm incision creates a tight fit with very little maneuverability for the needle. The result is to create
Descemet's folds or striae that may obscure visualization during phacoemulsification. Raising the infusion bottle will create
more inflow into the eye, but will not inflate the eye enough to decrease the striae. Similarly, decreasing the vacuum or
aspiration will have no effect on the striae because changes in the intraocular pressure are not responsible for the folds.
Only by enlarging the internal and external incisions and allowing more room for the phacoemulsification needle will the tight
fit be relaxed and the striae disappear.

Question 56 of 130
Which of the following statements is least accurate about the rubella syndrome?

The retina often demonstrates a "salt and pepper" appearance.

It often includes bilateral nuclear sclerotic cataracts.

It occurs when the mother is infected during the third trimester.

Virus-induced iridocyclitis may occur if all cortical and nuclear lens material is not removed during the initial cataract
surgery.
Please select an answer
Feedback: The rubella syndrome occurs when the mother is infected during the first trimester of pregnancy. Associated
findings include microphthalmos, shallow anterior chamber, a "salt and pepper" appearance of the fundus, hearing loss,
patent ductus arteriosis and other heart and valve abnormalities, and genitourinary defects. Histologically, the characteristic
finding in rubella cataract is retention of fetal cell nuclei in the cataractous lens nucleus. The rubella virus may be present in
the lens for up to 3 years after birth. All cortical and nuclear material must be removed during cataract surgery to avoid virus-
induced iridocyclitis.
Question 57 of 130
Toric intraocular lenses (IOLs) might rotate circumferentially within the capsule following in-the-bag implantation. How much
toric IOL rotation can occur before the IOL stops compensating for corneal astigmatism and begins to induce unwanted
astigmatism at another axis?

20 degrees

30 degrees

10 degrees

40 degrees
Please select an answer
Feedback: Toric intraocular lenses are currently under investigation. In the first phase of the U.S. FDA study, 124 toric
intraocular lenses were implanted. It was determined that those that remained within 30 of the intended axis reduced
astigmatism. Those that rotated 30-40 functioned refractively, like spherical intraocular lenses, and those that rotated 40
or more added cylinder to the postoperative refraction in another axis. In the unpublished FDA Phase I study of the STAAR
4203T toric intraocular lens, 93% of the intraocular lenses remained 30 off the intended axis. The overall average degree of
intraocular lens rotation was 8.9. The 2.0-D toric intraocular lens reduced astigmatism by an average of 1.0 D. Three toric
intraocular lenses rotated 40 and added an average of 0.37 D to the preoperative astigmatism.

Question 58 of 130
During phacoemulsification, a capsular break is noted, with vitreous in the anterior chamber. Nucleus removal is incomplete.
Which of the following steps should not be taken?

Consider conversion to an extracapsular procedure with use of a lens loop to remove the nuclear fragments.

Continue phacoemulsification because the phacoemulsification instrument efficiently cuts vitreous.

Inject viscoelastic beneath the nuclear remnant to keep it elevated.

Remove all vitreous from the anterior chamber using an automated vitrector and a low-infusion inflow rate.
Please select an answer
Feedback: When a capsular break is noted during phacoemulsification and nucleus removal is incomplete, strategies must
be undertaken to remove the nuclear fragment(s) and avoid the potential for a "dropped" nucleus. Visualization of the extent
of the capsular break or zonular dehiscence must be made. If the break is enlarging, it is best to stop and convert to an
extracapsular procedure. The wound should be enlarged and viscoelastic should be injected beneath the nuclear remnant to
keep it elevated. The nuclear fragment can be elevated above the plane of the iris with a cyclodialysis spatula, and a lens
loop can be inserted to remove the nuclear fragment(s). Expression-type techniques should be avoided. All vitreous can
then be removed from the anterior chamber using low-inflow vitrectomy to minimize extension of the capsular rent. If the
capsular rent is not enlarging, the surgeon should check for external pressure on the lid and globe by the speculum. The
infusion bottle should be lowered to minimize inflow and prevent extension of the break. The anterior chamber should be
swept with a cyclodialysis spatula to check for the presence of vitreous. Inefficient aspiration, either in the
irrigation/aspiration (I/A) or phacoemulsification mode, implies vitreous in the anterior chamber. Careful phacoemulsification
can proceed if the extent of the rent is visualized; a second instrument can be used to prevent loss of nuclear fragments
posteriorly through the capsular opening. Some surgeons advocate the use of a lens glide that is slid beneath the nucleus
under viscoelastic control and over the capsular opening, thus protecting against loss of nuclear material through the rent as
phacoemulsification proceeds. Following removal of the nucleus fragments, retained cortex can be removed. Cortical
material should be removed by pulling toward the break rather than away from it to avoid extending the opening. During
cortical removal, a secondary irrigating port can be used to minimize inflow. Cortical removal can also be done using a "dry
technique" in which the anterior chamber is filled with viscoelastic and an aspirating manual cannula is used to remove
residual cortical material. Use of the phacoemulsification needle to cut and remove vitreous is contraindicated as the
phacoemulsification needle is inefficient in cutting vitreous, and excessive vacuum can lead to significant traction on the
vitreous and the potential for a giant retinal tear.
Question 59 of 130
Three years after external beam radiation for unilateral retinoblastoma, a 7-year-old child presents with a cataract in the
same eye. The retinoblastoma proved to be radiation-sensitive, with complete regression of the tumor. The remainder of the
retina and retinal vasculature appear normal although the cataract obscures the view. The opposite eye is normal and is
emmetropic. Which of the following is the most appropriate form of treatment for the cataract?

Cataract extraction with use of an aphakic silicone contact lens

Cataract extraction with posterior chamber intraocular lens placed within the capsular bag

Cataract extraction with use of aphakic spectacles

Cataract extraction with use of an aphakic gas-permeable contact lens


Please select an answer
Feedback: The use of intraocular lenses in children remains controversial. However, as cataract techniques and intraocular
lenses have improved, indications for intraocular lens implantation in children have expanded. In the case of a unilateral
cataract in a 7-year-old child, aphakic spectacles are inappropriate because of the anisometropia that would occur
postoperatively. Because of handling difficulties and the potential for radiation-induced dry eye, contact lenses would
likewise be a poor choice. For this child, an intraocular lens is the best choice because anisometropia can be avoided as
well as the problems associated with handling and long-term wear of contact lenses. By age 7, the eye has reached adult
size and no modifications are necessary in calculating the intraocular lens power. Posterior capsular opacification is a
frequent postoperative occurrence, but in a cooperative child, is managed easily with Nd:YAG laser capsulotomy.

Question 60 of 130

Topical anesthetic agents produce analgesia in which of the following ocular structures?

Cornea and iris

Conjunctiva and ciliary muscle

Conjunctiva and cornea

Conjunctiva and dilator muscle


Please select an answer
Feedback: As the name implies, topical anesthesia is just that: topical. The ocular surfaces (conjunctiva and cornea) are
rendered analgesic by topically applied ocular anesthetic agents. The sensitive intraocular structures (the iris and ciliary
muscle) are innervated by the long ciliary nerves, and when these structures are manipulated at surgery, pain can be
induced. Pain can be caused either by direct manipulation of instrumentation (for example on the iris) or by simply
deepening the anterior chamber with the inflow of irrigation fluid, which causes the ciliary muscle to stretch. Some surgeons
have suggested that excellent analgesia of intraocular structures is obtained by the instillation of intracameral anesthetic
agents such as preservative-free 1% lidocaine.
Question 61 of 130
In a patient with cataract who is myopic and has a posterior staphyloma, which of the following methods of axial length
measurement would be most accurate?

Scanning slit topography

Applanation a-scan biometry

Partial coherence interferometry

Immersion a-scan biometry


Please select an answer
Feedback: In a patient with a myopic posterior staphyloma, the functional axial length may be difficult to obtain in part
because the fovea may not be located at the vertex of the staphyloma. A-scan biometry is based on the principle that sound
waves will be highly reflected when they encounter a surface perpendicular to the ultrasonic source, but if the fovea is not
located at this point, then the axial length measurement may be inaccurate. Partial coherence interferometry (Zeiss IOL
Master) is a light dependent technology based upon patient fixation, and can therefore theoretically provide a more
accuarate measurement of functional axial length.

Question 62 of 130
You are a busy and experienced cataract surgeon. In the first week following an uncomplicated phacoemulsification, you
discover that your patient is left with a spherical equivalent of +2.5 D. Your target refraction was plano. The patient's
preoperative axial length was 23.5mm on immersion axial biometry and the Ks were 44.0 D and 45.0 D, with normal corneal
topography and a normal slit lamp exam. What is the most likely explanation for this refractive surprise?

Inadvertent placement of the IOL in the sulcus instead of the capsular bag.

Use of the incorrect regression formula in the IOL power determination.

Placement of the wrong IOL due to confusion with another patient

Miscalculation of corneal power due to previous refractive surgery in the operated eye.
Please select an answer
Feedback: A hyperopic error of this magnitude is not likely due to a difference in the regression formula used, particularly in
a patient with relatively normal axial length and keratometry. With these biometric parameters, most common regression
formulas would predict similar IOL powers, especially for someone who is routinely doing cataract surgery without such large
inaccuracies. The inadvertent placement of an IOL in the sulcus instead of the capsular bag should create a small (<1.0)
myopic error, not a hyperopic one. Although previous refractive surgery can lead to a hyperopic error, this would be unlikely
given the patient's biometry, normal exam and corneal topography. Unfortunately, human error, through confusion of
patients or ACIOLs power calculations, is still common.
Question 63 of 130
While attempting to remove a nuclear fragment during routine cataract surgery, the anterior chamber suddenly collapses
completely, and the cornea folds downward onto the phaco tip. You see no bubbles in the irrigation line. What is the next
appropriate step in managing this situation?

Check that the irrigation bottle is not empty or that the irrigation has become unplugged from the phaco handpiece.

Call for the setup of anterior vitrectomy instrumentation.

Remove the phaco handpiece from the eye and reform the chamber with blanced salt solution.

Depress the foot pedal into poistion three so as to clear the phaco tip, restart irrigation, and reform the anterior
chamber.
Please select an answer
Feedback: This scenario occurs when irrigation is lost while attempting to vacuum or aspirate during phacoemulsification.
Although quite shocking when it occurs, a calm, thoughtful approach may prevent damage to intraocular structures. The two
most common causes of this problem are an empty irrigation bottle or the disengagement of the irrigation tubing from the
phaco handpiece. Some newer phaco machines can sense an empty irrigation bottle and prevent this complication, but an
unplugged irrigation port can still occur. These two variables should be checked immediately. In many cases, the immediate
restoration of irrigation will reform the anterior chamber and the case can proceed without secondary complications. Another
strategy would be to immediately reform the chamber with viscoelastic via a sideport incision. Depressing the foot pedal
would lead to further aspiration and vacuum and like cause further harm to intraocular structures. Although vitreous loss can
occur in this scenario, fortunately many times the poterior capsule will reamin intact if vacuum and aspiration are
immediately stopped. Removing the handpiece from the eye before reforming the anterior chamber can be risky because
there may be intraocular structures sequestered in the phaco tip including capsule, iris, or descemet's membrane.

Question 64 of 130
A patient with visually significant cataract is found to dilate poorly on preoperative examination. Which of the following is the
most likely cause of this poor dilation?

Pigment dispersion syndrome

Pseudoexfoliation syndrome

Atopic dermatitis

Hypertension
Please select an answer
Feedback: Pseudoexfoliation syndrome is a common disorder associated with the depostion of a fibrillogranular material on
the anterior surface of then lens and elsewhere in the anterior segment. With respect to cataract surgery, patients with this
condition may have zonular laxity, capsular fragility, and poor pupillary dilation.
Question 65 of 130
A patient who undergoes an uncomplicated cataract extraction sees no better postoperativley despite an excellent surgical
result. The macula appears normal on ophthalmoscopy and OCT scanning. Which of the following would be the best next
initial step?

YAG capsulotomy

Electroretinogram

MRI

Corneal topography
Please select an answer
Feedback: Poor acuity following cataract extraction can have many causes. The work up in determining the cause of poor
vision can include all of the tests listed above. However, the sequence of ancillary testing should take into account factors
including prevalence of disease and the invasiveness of the test. Because ocular surface disease and/or irregular
astigmatism can often limit visual acuity, a corneal topography is often a helpful initial test. It can also usually be performed
quickly and without significant risk or inconvenience to the patient.

Question 66 of 130
The ability of a viscoelastic material to transform under pressure from a gel to a liquid is known as which of the following?

Viscosity

Pseudoplasticity

Dispersiveness

Cohesiveness
Please select an answer
Feedback: Viscosity is a term that describes resistance to flow. Cohesiveness and dispersiveness describe the self-
adherence properties of a viscoelastic material.

Question 67 of 130
A 38 year old patient is referred to you from a dermatologist who is treating the patient for severe atopic dermatitis. What
type of cataract is this patient most likely to have?

Nuclear sclerotic cataract

Anterior subcapsular cataract

Posterior cortical cataract

Posterior polar cataract


Please select an answer
Feedback: Cataract can be found in up to 25% of patients with atopic dermatitis. These cataract are usually bilateral, central
anterior or posterior subcapsular opacities, with onset in the second or third decade of life. Although many will have minimal
symptoms, some will progress rapidly to visually signifcant cataracts requiring phacoemulsification surgery.
Question 68 of 130
You are attempting to engage a nucelar segment of mild to moderate density, but you are not able to obtain a high enough
vacuum to remove the piece despite increasing your ultrasound power to 80%. What is the term for this phenomenon that
best explains this difficulty?

Cavitation

Interferometry

Tip occlusion

Surge
Please select an answer
Feedback: This scenario describes difficulty in obtaining tip occlusion, which is required in order to generate the high
vacuum levels needed to grab, hold and maneuver nuclear pieces. If the ultrasonic power is too high for the density of the
cataract, obtaining tip occlusion will be difficult because of cavitation. The energey released by the cavitation created by the
ultrasoinc tip will emulsify too much nuclear material at the phaco tip, and limit the possibility of tip occlusion.

Question 69 of 130
While trying to remove nuclear segments during phacoemulsification, the followability of the nuclear pieces at the phaco tip
is poor, and small fragments of cataract are frequently seen in the anterior chamber. To mitigate this problem, one could
decrease which of the following parameters?

Bottle height

Aspiration/flow

Ultrasonic energy

Vacuum
Please select an answer
Feedback: When followability during segment removal is poor, this may be due to chatter as the ultrasonic repulsive force
overcomes the attractive forces of aspiration and vacuum. Increasing vacuum or aspiration can help decrease chatter, and
improve followability.

Question 70 of 130
A patient with severe Parkinson's disease presents with visually significant cataracts. You plan to operate under general
anesethsia due to excessive head movement. What is the best surgical approach to this case?

Pars plana lensectomy

Phacoemulsification through a superior scleral tunnel incision

Extracapsular cataract extraction through a superior incision

Phacoemulsification through a temporal clear corneal incision


Please select an answer
Feedback: Patients with Parkinson's disease have poor blink frequency. Because of this, they often suffer from severe
exposure keratopathy and poor epithelial wound healing. Because of this, an optimal cataract incision wold be under the lid
and covered by conjunctiva, as in a superior scleral tunnel incision. There is no contraindication to standard
phacoemulsification in these patients.
Question 71 of 130
Which of the following statements regarding radial keratotomy (RK) and photorefractive keratectomy (PRK) and regression
of effect is most accurate?

Regression of effect following refractive surgery for myopia implies that the refractive result shifts toward hyperopia.
This finding is more profound after RK.

Regression of effect following refractive surgery for myopia implies that the refractive result shifts toward myopia. This
finding is more profound after PRK.

Regression of effect following refractive surgery for myopia implies that the refractive result shifts toward hyperopia.
This finding is more profound after PRK.

Regression of effect following refractive surgery for myopia implies that the refractive result shifts toward myopia. This
finding is more profound after RK.
Please select an answer
Feedback: Both RK and PRK can cause some regression of effect. In RK this is usually during the first week and is a result
of edema around the incision wound. The edema contributes to increased midperipheral wound gaping, increased
midperipheral steepening, and central flattening. As the edema resolves, this flattening is reduced, decreasing the overall
effect and causing a shift toward myopia or away from hyperopia. In PRK, the epithelium is replaced over the first month
following the procedure. As the epithelial thickness is reestablished, the cornea steepens, thereby reducing the amount of
corneal flattening and shifting the refraction toward myopia. This shift is much more profound in PRK than it is in RK and is
particularly exaggerated when excess scarring occurs with subepithelial fibrosis, as seen in some PRK patients. Regression
of effect in PRK has been noted as long as 12 to 18 months following the procedure.

Question 72 of 130
What is the most important role to determine for refraction in the preoperative evaluation of the cataract patient?

Potential postoperative acuity

Risk of retinal detachment

Intraocular lens power

Best corrected visual acuity


Please select an answer
Feedback: The best corrected visual acuity should correlate with media. If the best corrected visual acuity and level of
media opacity do not match, other possible etiology for reduced visual acuity should be investigated.

Question 73 of 130
Which of the following is not a risk factor in developing an expulsive suprachoroidal hemorrhage intraoperatively?

Axial length greater than 26.0 mm

Traumatic cataract after history of remote trauma

Increased intraocular pressure

Intraoperative tachycardia
Please select an answer
Feedback: Intraoperative suprachoroidal hemorrhage is an uncommon event with a reported incidence of 0.05% to 0.8%.
Statistically significant risk factors for suprachoroidal expulsive hemorrhage in one age-adjusted analysis included
glaucoma, increased axial length, elevated intraocular pressure, generalized atherosclerosis, and elevated intraoperative
pulse. Attention to multiple preoperative and intraoperative ocular and systemic variables may allow identification of patients
at risk for suprachoroidal expulsive hemorrhage so that appropriate preventive measures may be instituted.

Question 74 of 130
Which of the following statements incorrectly describes the cornea?

The cornea flattens directly over any sutured incision.

The normal cornea steepens over any incision.

The central cornea steepens adjacent to tight limbal sutures.

Tissue removal produces corneal flattening over the site of tissue removal, whether traumatic or surgically induced.
Please select an answer
Feedback: The normal cornea flattens over any incision. This effect is attributable to gaping of the wound. A corneal incision
slices through the corneal lamella, and intraocular pressure spreads the edges apart. This increases the radius of curvature
of the cornea across the incision and flattens the cornea. The incision heals with a greater surface area, creating a micro-
wedge addition. This concept provides the basis for incisional refractive surgery. Tight corneal sutures have the effect of
flattening the cornea over the sutures but steepening the cornea central to the tight sutures. This insight is helpful in the
modification of astigmatism by cutting (removing) overly tight sutures. Tissue removal acts as a large relaxing incision with
increased gaping due to the loss of substance. Hence, the cornea will flatten over the site of the tissue removal.

Question 75 of 130
For chemical preparation of the eye prior to ophthalmic surgery, which of the following is most effective in decreasing the
bacterial flora of the conjunctiva without causing ocular surface toxicity?

A mild silver protein solution (Argyrol)

10% povidone-iodine (Betadine) solution followed by irrigation of the conjunctival fornix with saline

5% (half-strength) povidone-iodine (Betadine) solution

5% povidone-iodine (Betadine) scrub/soap


Please select an answer
Feedback: Half-strength povidone-iodine (Betadine) solution has been proven to be the most effective chemical preparation
for the eye prior to ophthalmic surgery. Povidone-iodine scrub/soap, on the other hand, is toxic to the epithelial surface and
causes a mild chemical burn. Irrigation of the fornix with saline increases bacterial flora counts of the conjunctiva by
liberating organisms from the conjunctival folds within the fornix. Argyrol solution does not have a significant antibacterial
effect on the conjunctiva. Chlorhexidine (Hibiclens) soap, commonly used as a hand-washing scrub prior to surgical
procedures, is also toxic to the ocular surface and should not be used in the chemical preparation of the eye prior to surgery.
Question 76 of 130

Which of the following disorders is associated with ectopia lentis?

Systemic lupus erythematosis

Homocystinuria

Turner's syndrome

Sturge-Weber syndrome
Please select an answer
Feedback: The differential diagnoses of simple ectopia lentis include homocystinuria, Marfan's syndrome, Weill-Marchesani
syndrome, hyperlysinemia, sulfate oxidase deficiency, congenital syphilis, trauma, Ehlers-Danlos syndrome, and exfoliation
syndrome.

Question 77 of 130
Which of the following statements does not accurately describe peribulbar anesthesia?

Supplemental anesthesia may be required.

Peribulbar anesthesia can be performed with needles that are shorter than those that are required for retrobulbar
anesthesia.

Globe perforation with peribulbar anesthesia is not possible.

Subdural injection is less common with a peribulbar technique than with a retrobulbar technique.
Please select an answer
Feedback: Peribulbar anesthesia has fewer serious complications than retrobulbar anesthesia and usually does not cause a
profound loss of vision. Because the needle does not enter the muscle cone, subdural injection is virtually impossible.
Because the peribulbar injection can be made in any of the four quadrants, perforation of the globe is possible and has been
anecdotally reported. A shorter needle is utilized to avoid a conal injection, and adequate anesthesia and akinesia is
achieved in most instances. Supplemental anesthesia by repeat peribulbar or retrobulbar injection is required in some
cases.

Question 78 of 130
Which of the following does not characterize phacolytic glaucoma?

A visible break in the lens capsule with inflammation around the lens material

A deep anterior chamber with circulating large, white, clumped cells

A unilateral red eye with diffuse corneal edema

An elevated intraocular pressure in the presence of a mature or hypermature cataract


Please select an answer
Feedback: Phacolytic glaucoma presents with a unilateral red eye, diffuse corneal edema, and elevated intraocular
pressure in the presence of a mature or hypermature cataract. Lens protein leaks from a mature or hypermature cataract
without a break in the lens capsule. The circulating large, white cells seen in phacolytic glaucoma represent foamy
macrophages that have ingested the leaking lens protein. Phacoanaphylactic endophthalmitis is a unilateral zonal,
granulomatous inflammation around lens material and is dependent on a ruptured lens capsule for its development.
Question 79 of 130
Indications for small-incision phacoemulsification surgery through a temporal clear-corneal incision under topical anesthesia
does not include which of the following?

A patient on warfarin (Coumadin) therapy for valvular heart disease

A patient with high axial myopia and increased axial length

An uncooperative patient with a history of psychosis

A monocular patient with glaucoma and a functioning superior filtering bleb


Please select an answer
Feedback: Temporal clear-corneal incisions under topical anesthesia minimize the chance of any bleeding complications,
including retrobulbar hemorrhage during a retrobulbar block and periocular ecchymoses during a peribulbar block. In
addition, conjunctival and scleral bleeding is avoided. Similarly, anesthesia risks are minimized in a monocular patient if
topical anesthesia is utilized, and manipulation of the sclera and conjunctiva is avoided with a clear-corneal incision in the
presence of a functioning filtering bleb. Inflammation is also minimized using a clear-corneal approach, which reduces
postoperative exacerbations of cicatricial ocular pemphigoid following cataract extraction. An uncooperative patient is a
contraindication to topical anesthesia because unwanted eye movement at inappropriate times may lead to serious
complications. In generat most cataract procedures can be done through a scleral tunnel incision; however, under certain
circumstances clear-corneal incisions can be advantageous.

Question 80 of 130

What is the most characteristic type of cataract in retinitis pigmentosa?

Posterior subcapsular cataract

Anterior polar cataract

Nuclear sclerotic cataract

Cortical spoke cataract


Please select an answer
Feedback: Posterior subcapsular cataracts are a frequent complication of retinitis pigmentosa (RP) and may be the primary
cause of poor central vision. The use of laser acuity meters or laser interferometry may help in differentiating the causes of
acuity loss. If careful selection criteria are used, cataract extraction with intraocular lens insertion may improve vision.
Unfortunately, in some patients, the retinal disease is too far advanced to warrant cataract surgery. The patient should be
cautioned that the potential benefit is to central vision without improvement of the peripheral visual field. The other types of
cataracts listed are not characteristic of RP.
Question 81 of 130
The centrally located cataract shown is associated with which of the following disorders?

Enhanced night-time contrast sensitivity

Atopic disease

A myopic shift in refraction

Early loss of near (reading) vision


Please select an answer
Feedback: The crystalline lens nucleus hardens and becomes more pigmented with age. These changes may be inferred
by the examiner and can appreciate that the nucleus has lost its lamellar structure and is becoming yellowed or amber in
color. The transition from a normally aging nucleus to a nuclear sclerotic cataract is often subtle. This transition may be
marked, not by easily observable morphologic changes, but by changes in visual function. The first functional change is
usually a myopic shift in refraction. This results in the "second sight" that enables some presbyopic patients to discard their
reading glasses. Blurring may be more accentuated at distance (particularly in dim illumination) than for near, even when
optimal spectacle correction is used. Monocular diplopia is also a frequent complaint.

Question 82 of 130
An 80-year-old woman underwent uneventful phacoemulsification and polymethylmethacrylate (PMMA) IOL implantation.
Preoperatively she was noted to have an axial length of 21.8 mm and pseudoexfoliation syndrome with a maximally dilated
pupil of 5.5 mm. A 4.5 mm capsulorhexis was performed at surgery and a 5.5-mm optic, 12-mm overall length IOL was
placed within the capsular bag. Three weeks after surgery, a contraction of the capsulorhexis opening to approximately 3.5
mm is noted. Which of the following treatments would be the most appropriate management?

Increase topical corticosteroid to reduce contraction

Perform multiple radial anterior Nd:YAG capsulotomies

Continue observation

Perform Nd:YAG posterior capsulotomy


Please select an answer
Feedback: Capsule contraction syndrome has been described in patients with previous uveitis, pseudoexfoliation
syndrome, advanced age, and retinitis pigmentosa. Its cause is thought to be an imbalance of the forces that affect
shrinkage of the capsular bag anatomy. Fibrotic contraction force is greater than zonular traction, which normally maintains
capsular bag diameter. These forces remain in opposition indefinitely, but most of the contracture seems to occur in the first
3 to 6 months. The earlier it is recognized, the earlier intervention may be accomplished to preserve zonular integrity and
reduce unwanted IOL shift or visual aberrations from the encroaching capsulorhexis edge. A course of continued
observation may result in further contraction and lead to decreased visual acuity. Posterior capsulotomy and topical
corticosteroids will not reduce contraction.
Question 83 of 130

On the first day following cataract surgery complicated by a posterior capsular rupture, you find a fragment of retained
crystalline lens. Which of the following findings would be sufficient to recommend a return to the operating room for its
removal?

Decreased vision

Anterior chamber cells

Retention of a large nuclear fragment

Elevated intraocular pressure


Please select an answer
Feedback: When retained lens material follows a complicated cataract surgery, many patients require a return to the
operating room for removal of the retained crystalline lens material, often by a surgeon skilled in pars plana vitrectomy
techniques. However, in cases in which small, cortical lens fragments are retained, topical treatment for increased
inflammation and/or elevated intraocular pressure and close observation may be preferred. Factors that support the removal
of retained lens fragments include large size, nuclear density, associated retinal tears or detachment, associated or possible
endophthalmitis, and elevated intraocular pressure or inflammation not responsive to medical therapy. In this case, it is only
the first postoperative day, so inflammation, elevation in IOP, and decreased vision may be normal or caused by other post-
operative factors (corneal edema, viscoelastic, prolonged surgical time). The symptoms and signs may improve over the
next few days with aggressive, topical medical therapy. The presence of a large retained nuclear fragment, however, likely
warrants timely referral and removal.

Question 84 of 130

An 18-year-old patient complains of a decrease in vision for the past several months. Examination shows the upper and
lower eyelids to be thickened. The lateral canthi are fissured and inflammed. Slit-lamp examination demonstrates an anterior
subcapsular cataract in one eye. In addition, the patient complains of multiple non-lid skin lesions that are dry,
erythematous, and very pruritic. Which of the following conditions is most likely represented by this constellation of ocular
and non-ocular findings?

Herpes simplex infection

Atopic disease

Varicella infection

Systemic lupus erythematosus


Please select an answer
Feedback: Anterior subcapsular cataracts occur in approximately 10% of patients with atopic disease. The skin lesions of
atopic dermatitis are dry, erythematous, and pruritic. Scratching leads to excoriation, weeping, scaling, and crusting. The
face and eyelids can be affected, especially in more severe cases. Corneal findings include punctate keratitis, marginal
ulceration, vascularization, and stromal opacification. The skin rash described here is not characteristic of lupus, in which an
erythematous rash appears on the nose and cheeks in a butterfly distribution. Herpes simplex is characterized by dendritic
keratitis and a vesicular eruptions fo the skin. Acute varicella infection is not typically associated with anterior subcapsular
cataract.
Question 85 of 130

On the 6th postoperative day following an uncomplicated cataract surgery, your patient presents with decreased vision, pain,
and a hypopyon. What would be the most likely cause?

Endophthalmitis due to gram-positive coagulase-negative bacteria

Undertreated post-operative uveitis

Toxic anterior segment syndrome (TASS)

Endophthalmitis due to candida albicans


Please select an answer
Feedback: This is a classic clinical presentation for acute postoperative endophthalmitis. Although TASS can also present
with similar signs and symptoms, TASS is usually seen on the first postoperative day. Acute postoperative endophthalmitis
can be caused by various organisms including gram-negative bacteria and fungi, however the most common infecting
organisms are coagulase-negative gram-positive bacteria.

Question 86 of 130
Which of the following is a deficiency common to all glare testing methods?

No control of incident glare angle to visual axis

No control of target contrast

No control of illumination intensity

No control of pupil size


Please select an answer
Feedback: Although factors that affect glare perception, such as illumination stimulus intensity, target contrast, and incident
glare angle, can all be adjusted or controlled during glare testing of patients, pupil size cannot be controlled for without
pharmacologic intervention. Once pharmacologic therapy is used to fix pupil size, glare testing may not accurately reflect the
patient's symptoms under physiological conditions. This is because the pupil diameter (and therefore glare characteristics
from non-uniform lens opacities) is dynamically determined, and variations from moment-to-moment changes in size affect
glare perception.
Question 87 of 130
A 32-year-old woman presents with 20/40 vision in both eyes. With refraction she is correctable to 20/25 bilaterally, but has
the lens opacity shown in the figure. If this cataract progresses, which one of the following complications is this patient at
greater that usual risk to develop?

Intraoperative capsular rupture

Postoperative uveitis

Intraoperative zonular dialysis

Postoperative choroidal detachment


Please select an answer
Feedback: The figure depicts a posterior polar cataract, one of the more common types of visually symptomatic congenital
cataracts. Posterior polar cataracts are dense white opacities axially positioned on the posterior capsule. The central portion
typically is circular and thick and has a characteristic concentric whorl-like appearance. Posterior capsular rupture is a
particularly common complication when performing cataract surgery in these patients. Ciliochoroidal detachments are seen
immediately following cataract surgery, but are not more commonly associated with this type of cataract. Choroidal
detachments are attributed to ocular hypotension; this type of postoperative choroidal detachment usually subsides within 3
weeks and requires no treatment. Late postoperative choroidal detachments occurring 7 to 21 days after surgery are
frequently associated with a persistent wound leak, delayed wound healing, or rupture of an inadequately healed wound; as
a rule, the first sign is some shallowing of the anterior chamber. Zonular dialysis may be associated with prior ocular trauma
or the ectopia lentis syndromes, but not with posterior polar cataract. Postoperative uveitis is no more common with
posterior polar cataracts than with any other type of cataract.

Question 88 of 130
Which of the following statements best describes the anterior chamber depth considerations in highly myopic patients
undergoing cataract surgery?

The anterior chamber depth is deeper than average and remains constant in depth.

The anterior chamber depth is deeper than average and is prone to fluctuating in depth.

The anterior chamber depth is shallower than average and remains constant in depth.

The anterior chamber depth is shallower than average and is prone to fluctuatind depth.
Please select an answer
Feedback:

Highly myopic eyes typically have deeper anterior chambers than hyperopic or emmetropic ones. Even if a deep chamber is
not obvious at slit-lamp examination, once the anterior chamber is filled with viscoelastic or the irrigating fluid from a
phacoemulsification probe, the anterior chamber often deepens significantly. However, the chamber depth in these eyes can
greatly vary with intraocular pressure changes during phacoemulsification. Irrigating posterior to the iris plane can
sometimes cause a sudden, dramatic shallowing of the chamber with associated pupillary miosis. This can require removal
of the instrument and reinsertion of the irrigating probe to restore the chamber to its original depth. When a very deep
chamber is not desirable, the chamber depth can be reduced in a controlled fashion using specialized techniques.
Question 89 of 130
You are asked to examine a 54-year-old engineer who would like to be able to work on his computer without glasses. His
refractive error is -6.50 D sphere OU. He is otherwise a good candidate for refractive surgery. He states that he would
accept "thin" glasses for other activities. What would the most appropriate optical treatment for his needs be?

Fully correct him for emmetropia with Lasik.

Correct all but 1D of the myopia in each eye.

Make him slightly hyperopic in one eye and myopic in the other.

Fully correct him for emmetropia with PRK.


Please select an answer
Feedback: Given the choices, this patient would be best served by undercorrecting him by a diopter in each eye. Fully
correcting his refractive error may produce the best uncorrected distance acuity, but it will not likely please this patient given
his desire to use the computer without glasses. Even if he has some residual accomodative range, this will dissipate over
time, leaving him spectacle-dependent for near activities if he has no residual myopia. Although monovision may be an
option, making him slightly hyperopic in one eye and myopic in the other would create anisometropia and would be poorly
tolerated by this myopic patient.

Question 90 of 130

An intraoperative technique using ancillary surgical devices is shown in the figure. This technique would be most
advantageous in patients with which of the following?

Phacolytic glaucoma

Exfoliation syndrome

A dislocated crystalline lens

Positive vitreous pressure


Please select an answer
Feedback: The figure demonstrates the use of polypropylene iris retractor hooks, which have proven to be beneficial in
increasing the pupillary diameter, thereby facilitating cataract extraction. Studies have shown that pupils in exfoliation
syndrome dilate submaximally. This is presumably related to iris infiltration and fibrosis from the exfoliation. The other three
conditions listed are not associated with submaximal pupillary dilation. Other methods of increasing the pupillary diameter
for facilitation of cataract extraction include radial sphincterotomies and sector iridectomies.
Question 91 of 130

A patient with advanced primary open angle glaucoma (POAG) and a cataract chooses to have a combined cataract and
glaucoma procedure rather than sequential surgery. Which of the following glaucoma procedures would be the best choice
to combine with cataract surgery?

Transscleral cyclophotocoagulation (TCP)

Trabeculectomy with mitomycin C

Goniotomy

Peripheral iridectomy
Please select an answer
Feedback: Glaucoma procedures that are commonly performed with cataract surgery in POAG include trabeculectomy,
seton-valve placement, and endocyclophotocoagulation (ECP). Primary goniotomy is usually performed for children with
congenital glaucoma. A peripheral iridectomy may be performed as part of another glaucoma procedure (eg,
trabeculectomy), but by itself it is not a treatment for POAG. TCP is usually reserved for eyes with end stage glaucoma and
little visual potential.

Question 92 of 130
Which of the following factors is the most likely cause of a large error in calculating the intraocular lens (IOL) power?

Lens A-constant

Corneal diameter

Axial length (AL) measurement

Anterior chamber depth


Please select an answer
Feedback:

Because AL is the most influential variable in IOL power, small errors in AL measurement can lead to large errors in IOL
selection. In particular, AL measurement by contact A-scan ultrasonography may lead to IOL calculation errors because the
ultrasound probe can inadvertently compress the cornea and lead to an underestimation of AL. Although errors in A-
constant or anterior chamber depth can also lead to IOL errors, the magnitude of the effect of these errors is usually small.

Question 93 of 130
What is the relationship regarding endothelial cell loss and the application of phacoemulsification for crystalline lens
removal?

Endothelilal cell loss is equivilent for scleral incisions and corneal incisions

Endothelial cell loss is reduced by in-situ (within the lens capsule) phacoemulsification compared to anterior chamber
phacoemulsivication

Endothelial cell loss is equivilent for superior and temporal clear-corneal incisions

Endothelial cell loss is increased by in-situ (within the lens capsule) phacoemulsification compared to anterior chamber
phacoemulsification
Please select an answer
Feedback: Studies of endothelial cell loss with phacoemulsifications have reported the following averages: anterior chamber
20%; posterior chamber 14%; in-situ 7%; intercapsular 4%. Extracapsular extraction has also reported to an average 7% cell
loss, approximately that of in-situ phacoemulsification. The reason most widely accepted to explain the decreased cell loss
with in-situ phacoemulsification is that the damaging ultrasonic energy is farther away from the endothelium. It was also
thought that in-situ phacoemulsification was so far away from the endothelium that the previously known linear relationship
between ultrasound time and cell loss seen with both anterior and posterior chamber phacoemulsification no longer applied
with in-situ techniques. Indeed, some cases of 15 and 20 minutes of ultrasound were anecdotally reported to have had no
cell loss when all the ultrasound energy was confined below the anterior capsule, such as in the chip-and-flip technique.
Superior clear-corneal incisions have demonstrated more cell loss (15% average) than temporal incisions (7% average),
theoretically because superior incisions are closer to the central endothelium than the more peripheral and more posterior
temporal incisions.

Question 94 of 130
How does a clear corneal cataract incision alter corneal curvature?

Flattens in the meridian of the incision

Flattens 90 degrees to the incision

Steepens in the meridian of the incision

Flattens the central cornea


Please select an answer
Feedback: Clear corneal cataract incisions can function like arcuate incisions used to correct astigmatism. They flatten the
cornea in the meridian of the incision, and by virtue of "coupling," steepening the cornea 90 degrees away. The amount of
effect is related to 2 variables: the size of the optical zone and the length of the incision. Because of these factors, surgeons
who perform clear corneal cataract procedures may choose to make the incision on the steep meridian and choose a PMMA
lens rather than a foldable lens for cases of high preoperative cylinder. Temporal incisions less than 3.5 mm (foldable lOLs)
that are within 0.5 mm to 1.0 mm of the limbus can usually correct no more than 1.25 D. Adding a single matched peripheral
arcuate incision 180 degrees away can correct to 2 or more diopters of astigmatism.

Question 95 of 130

A 65-year-old wishes cataract surgery for recent decreased vision. He reports that he has had bilateral radial keratotomy
years ago. What would you recommend prior to surgery?

Refer the patient for a deep anterior lamellar keratoplasty (DALK)

Determine the patient's true corneal power

Perform preoperative photorefractive keratectomy (PRK)

Refer the patient for riboflavin-assisted collagen cross-linking


Please select an answer
Feedback: Standard keratometry following radial keratotomy (RK) is notoriously inaccurate in estimating the true corneal
power. In most cases, the keratometry is steeper than the known refractive effect following the radial keratotomy. Following
RK, the refractive effect in diopters is usually greater than the change in keratometry. Thus, if measured keratometry is
entered in the intraocular lens (IOL) calculation, the predicted power of the IOL for emmetropia will usually be
underestimated, leading to an undesirable hyperopic result. Although video keratography may in some cases provide more
accurate readings than the actual keratometry, patients should be cautioned that because of their previous radial
keratotomy, IOL calculations may be inaccurate, necessitating additional refractive correction, or in some cases, IOL
exchange. PRK, DALK, and collagen cross-linking are usually not necessary or indicated in patients with stable RK incisions
undergoing cataract surgery.
Question 96 of 130
Years following uncomplicated phacoemulsification with sulcus placement of a 3-piece posterior chamber lens, a healthy
adult patient notes intermittent obscurations of vision. Examination during an episode of decreased vision demonstrates a
microhyphema and elevation of intraocular pressure to 30 mm Hg. Transillumination defects of the iris are visible on
retroillumination. What is the most likely diagnosis?

Pseudophakic iris chafing syndrome

Iris nevus syndrome

Peripheral uveitis

Neovascularization
Please select an answer
Feedback: Posterior iris chafing by the loop or the optic portion of sulcus-fixated posterior chamber lens implants may
cause a spectrum of disorders, including iris-pigment epithelial defects, pigment dispersions with or without intraocular
pressure elevation, intermittent microhyphemas with transient visual obscurations, and uveitis-glaucoma-hyphema (UGH)
syndrome. It appears that secondary pigmentary glaucoma is more likely with planar haptic designs than with angulated
haptics. Optic and haptic materials may play a role in the development of the disorder. Implantation of both supporting
haptics of the implant within the capsular bag is suggested to prevent posterior iris chafing. Iris transillumination defects are
not normally seen in peripheral uveitis or in the iris nevus syndrome, where the iris is studded with multiple nevi.

Question 97 of 130

Following an intraoperative capsular tearing, what would be your preferred use of the one-piece acrylic intraocular lens (IOL)
you had selected to implant?

Placed in the ciliary sulcus if a tear in the capsulorhexis occurs

Should be sutured to the sclera

Placed in the iridocorneal angle

Sutured to the iris


Please select an answer
Feedback:

Single-piece acrylic IOLs are not ideal for suclus placement or scleral suturing following complicated cataract surgery. In
such cases, many surgeons elect to use a 3-piece lens with ciliary sulcus placement. In cases where sulcus support is
inadequate, iris fixation with McCannell sutures can be considered. Most scleral-sutured lenses have a specialized haptic
design with eyelets to facilitate suturing. These lenses are not sized or rigid enough for placement in the iridocorneal angle.
Question 98 of 130
Bilateral anterior lenticonus is often associated with which of the following syndromes?

Lowe syndrome

Down syndrome

Alport syndrome

Congenital rubella
Please select an answer
Feedback: Alport syndrome is a hereditary renal disease generally transmitted in an autosomal dominant fashion. The
disorder is more severe in males, and nerve deafness is a prominent feature. The most significant ocular abnormalities are
anterior lenticonus and anterior polar cataracts. The most common ocular complication of Lowe syndrome is the presence of
congenital cataracts and posterior lenticonus. Lenticonus is not a feature of either rubella or Down syndrome.

Question 99 of 130
The day after an uncomplicated phacoemulsification a patient's intraocular pressure is 32 mm Hg with open iridocorneal
angle. The patient has never had an IOP measured higher than 21 mm Hg. How would you manage his elevated intraocular
pressure?

Burp the sutureless clear corneal incision

Prescribe oral acetazolamide

Perform a laser peripheral iridotomy

Prescribe an ocular hypotensive drug


Please select an answer
Feedback: Temporary postoperative elevations in intraocular pressure (IOP) are not uncommon following cataract surgery.
These IOP spikes can be managed with topical ocular hypotenive drugs alone. Oral therapy is not usually required since
theses IOP rises are usually mild and temporary. Because the mechanism of IOP elevation is usually due to compromised
outflow due to retained viscoleastic, laser iridotomy will not likely be effective. In severe cases of IOP elevation, some
physicians may elect to decrease the IOP by burping the paracentesis incision, but burping the main incision is not
recommended.

Question 100 of 130

Sunrise syndrome or superior subluxation of an intraocular lens is caused by what anatomical feature?

Displacement of a haptic by lens pearls

Contraction of the lens capsule

Peripheral contraction of the lens capsule

One lens haptic in the capsular bag and one lens haptic in the ciliary sulcus
Please select an answer
Feedback: Sunrise syndrome occurs when the IOL optic is displaced superiorly, out of the visual axis. One cause of this
problem is placement of the inferior haptic within the bag and placement of the superior haptic within the sulcus. With
contraction of the capsular bag, the inferior haptic is pushed upward because there is no resistance from the capsule
superiorly. Another cause of this problem is inferior zonular disinsertion with an in-the-bag placement of a flexible-haptic
posterior chamber lens. The fibrosis of the capsule drags the optic superiorly, until the edge of the optic is in or above the
visual axis. Finally, a disruption of the superior zonules when the inferior haptic is in the bag and the superior haptic is
through the disinsertion will also cause a sunrise syndrome as contraction of the bag forces the superior haptic through the
disinsertion. Lens pearls cannot displace a haptic enough to cause sunrise syndrome. Peripheral capsular contraction
would not be sufficient to displace lens as shown. A small-optic IOL may decenter slightly within an intact contracting
capsule, but not to the degree seen in the figure.

Question 101 of 130

During the performance of cataract surgery on a high myope, your view becomes limited due to numerous corneal striae
caused by the very deep anterior chamber and the steep angle of your intraocular instruments. What maneuver would be
the preferred option to improve your view?

Convert to small-incision ECCE

Debride the surface epithelium that is obscuring your view

Use a second instrument to lift the iris from the anterior lens capsular surface

Raise the bottle height


Please select an answer
Feedback:

The best approach to this problem would be to shallow the anterior chamber so that the angle of instrumentation is less
steep. This can be accomplished by using a second instrument (eg cyclodialysis spatula) to slowly elevate the iris from the
surface of the anterior capsule. This is a controlled way to adjust the lens iris diaphragm depth to a level that suits one's
comfort level. In contrast, raising the infusion pressure would deepen the anterior chamber and worsen the situation.
Converting to small incision ECCE would shallow the chamber but would not be preferred as it would create multiple
pathways of fluid loss. Debrieding the corneal surface is useful to manage corneal epithelial edema, but would not improve
the vision from corneal traction striae.

Question 102 of 130

When considering lens surgery for ectopia lens, for which disorder should you be concerned about the systemic risk of a
hypercoaguable state?

syphilis

homocystinuria

Marfan's syndrome

hyperlysinemia
Please select an answer
Feedback:

Patients with ectopia lentis may have other systemic findings depending on the syndrome or disease associated with the
lens dislocation. Patients with Marfan's syndrome are at risk for coarctation of the aorta and cardiac valvular disease.
Patients with homocystinuria have a hypercoaguable state and are at increased risk for thrombotic events. No bleeding
predisposition is associated with syphilis or hyper lysinemia.
Question 103 of 130
For an preoperative eye with normal keratometry values and hyperopia of +2.50 D, what axial length would you expect on A-
scan biometry?

greater than 26.0 mm

less than 23.0 mm

greater than 23.0 mm

less than 20.00 mm


Please select an answer
Feedback: For an eye with normal keratometry values, one can estimate the relationship between axial length and
preoperative refraction, assuming a minimal myopic contribution due to cataract. For high myopes, one would expect a
relatively long axial length (greater than 26.0mm). For a high hyperope, one would expect a relatively short axial length (less
than 20.0mm). For a low hyperope, one would expect an axial length less than 23.0mm.

Question 104 of 130


Which of the following conditions has the strongest association with microspherophakia?

Weill-Marchesani syndrome

Lowe syndrome

Marfans syndrome

Alports syndrome
Please select an answer
Feedback: Microspherophakia can rarely be associated with all of the conditions listed; however, it is most often and
characteristically seen in patients with Weill-Marchesani syndrome. Features of this autosomal recessive syndrome include
short stature, short and stubby fingers, and reduced joint mobility.

Question 105 of 130


Which of the following actions is not helpful in the treatment of posterior infusion (infusion misdirection) syndrome?

Aspiration of fluid through the pars plana

Infusion of IV mannitol

Raising of the infusion bottle

Gentle posterior pressure on the lens


Please select an answer
Feedback: Posterior infusion syndrome is used to describe an intraoperative occurence when fluid infused into the anterior
chamber becomes inadvertently misdirected into the vitreous cavity. This causes marked forward movement of the lens and
shallowing of the anterior chamber. Applying pressure to the posterior lens may change the predominant posterior flow and
successfully reduce the condition. Infusion of IV mannitol and aspiration of posterior fluid through the pars plana will reduce
the pressure in the vitreous, also relieving the posterior pressure. Raising of the infusion bottle will aggrevate the problem
by increasing posterior flow.
Question 106 of 130
During preoperative cataract evaluation, a symptomatic retinal flap-tear is discovered. What would be the most appropriate
management?

laser demarcation prior to cataract surgery

cataract surgery with intra-operative photocoagulation

laser demarcation immediately after cataract surgery (improved view)

careful cataract surgery


Please select an answer
Feedback: A symptomatic retinal flap-tear is a risk factor with an ongoing high risk for retinal detachment. Such a tear
should be treated prior to proceeding with elective cataract surgery.

Question 107 of 130


Which intraocular lens power-calculation formula would be most appropriate for a patient with high axial hyperopia?

SRK II

Colenbrander

Hoffer Q

SRK
Please select an answer
Feedback: The Colenbrander formula is an early generation theoretical formula which is no longer commonly used due to
the accuracy of empiric regression formulas. Of the modern regression formulas, the Hoffer Q is considered the most
accurate in the setting of very short eyes.

Question 108 of 130


During preoperative cataract evaluation, a patient is found to have 2.5 D of with-the-rule astigmatism, but the keratometric
measurements show only 1.5 D of with-the-rule astigmatism. At the time of cataract surgery, which one of the following
options would minimize the postoperative astigmatism?

a 3.5 mm superior straight scleral incision with paired 8 mm limbal relaxing incisions (LRIs) in the 180-degree meridian

a 6.0 mm superior scleral frown incision secured with a single horizontal suture

a 3.0 mm temporal clear cornea incision with implantation of a toric IOL with +3 D of cylinder oriented with axis at 180-
degrees

a 3.0 mm temporal clear cornea incision with paired 6 mm limbal relaxing incisions (LRIs) in the 90-degree meridian
Please select an answer
Feedback: Because this patient has with-the-rule corneal astigmatism, procedures that decrease astigmatism in the 90
degree meridian will lead to the least post-operative astigmatism. Limbal relaxing incisions (LRIs) or a toric IOL placement in
the 180 degree meridian will increase post-operative with-the-rule astigmatism. Although a wound in the 90 degree meridian
may decrease astigmatism somewhat, this effect is mitigated by the placement of a suture or a frown wound configuration.
Question 109 of 130
Which of the following tests would be most helpful in determining the etiology of lens subluxation in a patient with ectopia
lentis?

cardiac echo

abdominal CT scan

HLA A-29

serum lysozyme
Please select an answer
Feedback: Ectopia lentis is commonly associated with several conditions including Marfan's syndrome, homocystinuria
and aniridia. It can also be see in association Ehlers-Danlos syndrome, sulfite oxidase deficiency and hyperlysinemia.
Because patients with Marfan's syndrome frequently have cardiac anomalies, a cardiac echo may be helpful. The other tests
listed are not specifically helpful in identfying conditions associated with ectopia lentis.

Question 110 of 130

In determining the axial length using ultrasound biometry, which eye condition result in the slowest ultrasound velocity?

Aphakia

Silicone oil

Post-vitrectomy

Phakia
Please select an answer
Feedback: Ultrasound biometry is used to measure axial length based on the speed that sound travels through intraocular
media. The speed of sound in silicone oil is slower than the speed in aqueous, vitreous or lens. Adjustments must be made
to axial length calculations for eyes filled with silicone oil to avoid large errors in axial length measurement.

Question 111 of 130


Which of the following is a common complication of iridodialysis during cataract surgery?

Permanent elevation of intraocular pressure (IOP)

Postoperative retinal detachment

Sympathetic ophthalmia

Intraocular bleeding/hyphema
Please select an answer
Feedback: There is no known association between iridodialysis and retinal detachment or sympathetic ophthalmia.
Although there can be a postoperative elevation in IOP in these cases, permanent elevation of IOP would not be expected.
Question 112 of 130
During hydrodissection of a routine case, the iris prolapses, the chamber shallows, and the eye becomes very firm. What is
the most likely cause?

Zonular dialysis and vitreous prolapse

Expulsive choroidal hemorrhage

Ciliary body edema

Capsular block syndrome


Please select an answer
Feedback: During hydrodissection, the fluid that passes behind the lens can cause the lens to more anteriorly, shallowing
the chamber. If there is a large nucleus or small capsulorhexis, this anterior lens movement can occlude the capsulorhexis,
preventing fluid from escaping around the lens equator. If more fluid is injected without releasing the fluid behind the lens,
the eye can become quite firm. It will often remain firm until the either the capsular block is broken or the posterior capsule
ruptures. This clinical scenario is sometimes confused with an expulsive choroidal hemorrhage. Neither ciliary body edema
or zonular dialysis would explain these findings.

Question 113 of 130


Following implantation of an intraocular lens (IOL), a 0.25-mm error in the IOL position would result in the greatest post-
operative refractive error in what condition?

Intraoperative floppy-iris syndrome

Nanophthalmia

High axial myopia with posterior staphyloma

Prior retinal detachment


Please select an answer
Feedback: The greatest post-operative refractive error would result from an IOL placed in a short eye. Intraoperative floppy-
iris syndrome would not increase or reduce IOL location refractive effect. Prior retinal detachment would similarly not affect
refractive effect of IOL location. However, if the retinal detachment were repaired with a scleral buckle, which increased the
axial length, it would reduce the refractive error induced by IOL malposition.

Question 114 of 130


Compared to primary IOL placement, what complication is increased following sulcus-placed piggyback IOL insertion?

Hyphema

Corneal edema

Interlenticular opacification

Retinal detachment
Please select an answer
Feedback: Piggyback IOL insertion appears to carry the same risk of retinal detachment, corneal edema, and hyphema as
standard IOL placement. However, the interface between to the two IOLs can become opacified. This opacity is not readily
treated with the YAG laser and usually requires a return to the OR to correct. Unfortunately, interlenticular opacification can
also recur.
Question 115 of 130
What prophylaxis has been demonstrated to be most helpful in preventing postoperative endophthalmitis?

Preoperative administration of topical antibiotic

Application of 5% povidone-iodine solution to the ocular surface

Barrier draping of the lashes

Addition of antibiotic to the irrigating solution


Please select an answer
Feedback: Other than the direct injection of intracameral cefuroxime into the anterior chamber at the conclusion of cataract
surgery, application of povidone iodine is the only other method of prophylaxis that has been demonstrated to be effective in
preventing endophthalmitis.

Question 116 of 130


The preferred placement and composition of secondary intraocular lenses (IOL) may depend upon individual patient
situation. For what situation is the lens choice most appropriate?

Acrylic one-piece IOL in the ciliary sulcus

Anterior-chamber IOL in an eye with aniridia

Silicone IOL in an eye with active uveitis

Acrylic IOL in an eye that may require silicone oil


Please select an answer
Feedback: Acrylic IOLs, unlike silicone IOLs, are appropriate to place in patients likely to need silicone oil. Droplets of liquid
silicone are adherant to IOLs made of silicone rubber, resulting in poor optical clarity. Acrylic one-piece IOLs are not
appropriate for sulcus placement due to the size of the haptics and the absence of optic vaulting. PMMA or acrylic IOLS are
preferred in patients with uveitis. Anterior-chamber IOLS should be avoided in patients with aniridia.

Question 117 of 130


For a patient with inactive uveitis and a cataract, what would be the preferred incision for cataract surgery?

The surgeon's choice

Sutured temporal clear cornea

Sutureless temporal clear cornea

Temporal scleral tunnel


Please select an answer
Feedback: For cases of uveitis with no corneal involvement, there is no evidence to suggest that one cataract incision is
superior to another. Some cases of uveitis are associated with glaucoma secondary either to inflammation or steroiduse. In
these instances, it may be prudent to avoid a superior scleral tunnel incision, because superior subconjunctival scarring may
affect future filtering surgery.
Question 118 of 130
Prior to cataract surgery, a patient reports an allergic reaction to Novocaine (procaine), an ester-containing anesthetic agent.
What other local anesthetic should also be avoided?

Tetracaine

Mepivacaine

Lidocaine

Bupivacaine
Please select an answer
Feedback: If there is documented allergy to procaine, other ester-linked local anesthetics should be avoided. These include
tetracaine, benzocaine, and butacaine. Lidocaine, bupivacaine, and mepivacaine are all amide-linked local anesthetics.

Question 119 of 130

A 27-year-old high myopic patient develops a cataract in the left eye following blunt trauma. His refraction is -6.00 sph OD
yielding 20/20 and -9.00 sph OS yielding 20/70. Which of the following is next preferred step in this patient's management?

Cataract surgery left eye with a refractive target of -6.00 D utilizing a multifocal IOL

A contact lens trial prior to cataract surgery

Cataract surgery left eye with a refractive target of plano utilizing a multifocal IOL

Cataract surgery left eye with a refractive target of plano utilizing a monofocal IOL
Please select an answer
Feedback: This patient needs cataract surgery in the left eye, but the post-operative target refraction presents special
problems. Following trauma, zonular integrity may be a concern, which is a relative contraindication for some types of
multifocal IOLs. Since there is no mention of a cataract in the non-traumatized right eye, one cannot assume that cataract
surgery will be needed in that eye soon. A post-operative refractive target of plano would result in anisometropia of 6 D
between eyes, an intolerable magnitude. If a target of -6 D is used and achieved, then the patient will likely require
additional myopic refractive correction in the left eye when, in the future, cataract surgery is performed in the right eye. The
preferred choice for this patient is to have a contact lens trial to determine that they can tolerate correction for a 3 to 6 D
anisometropia that will result from left eye cataract surgery. Then a plano target may be considered for the left eye.

Question 120 of 130


What condition is associated with the development of cataract?

Primary open-angle glaucoma

Choroidal neovascularization

Systemic use of a nonsteroidal anti-inflammatory drug

Retinitis pigmentosa
Please select an answer
Feedback: Retinitis pigmentosa is associated with posterior subcapsular cataracts. Care should be taken to evaluate these
patients for concomitant cystoid macular edema, which may also be a cause of decreased central acuity in patients with
retinitis pigmentosa. Topical steroids are a known risk factor for the development of cataract, but NSAIDs do not carry any
similar risk. Choroidal neovascularization and open angle glaucoma do not cause cataracts.
Question 121 of 130
In a myopic eye with a posterior staphaloma, what is the most reliable method for determining the axial length?

Optical coherence biometry

Applanation A-scan biometry

Vector A/B-scan biometry

Immersion A-scan biometry


Please select an answer
Feedback: Because ultrasonic biometry, both applanation and immersion, is based on the principle of reflection and
perpendicularity, eyes with posterior staphymomas present unique problems. The location of the best A-scan spike, i.e., the
point at which the ultrasound waves are reflected from a perpendicular surface, may be coming from the apex of the
staphyloma rather than from the fovea. Because optical coherence biometry is aligned on visual fixation, the measurement
can be more reliably attributed to the anatomical fovea, avoiding concerns about measuring a staphyloma's apex.

Question 122 of 130


When performing applanation or immersion A-scan biometry, what waveform characteristic should be optimized to assure
the most accurate globe length measurement?

Equally spaced peaks

Peaks of equal height

High and steeply rising

Peaks of equal width


Please select an answer
Feedback: A-scan biometry depends on identying the ocular interfaces (e.g. vitreous/retina) that are most perpendicular to
the ultrasonic source/sensor. This ensures that the maximum globe diameter is measured. Surfaces that are perpendicular
to the ultrasonic wave source will have the most reflectivity for a given acoustic interface. These highly reflected waves are
then measured by the sensor as high and steeply rising peaks. Alternativlely, ultrasonic waves that are reflected obliquely by
non-perpendicular surfaces will produce shorter, blunter spikes on A-scan biometry.

Question 123 of 130


Unless a special correction is made, what clinical condition will produce a falsely long axial length by A-scan biometry?

Prior vitrectomy

Scleral buckle from retinal detachment repair

Prior keratorefractive surgery

Silicone oil in the vitreous cavity


Please select an answer
Feedback:

A-scan ultrasound measures axial length based on the reflected velocity of sound waves traveling through the eye. When
the vitreous is replaced by silicone oil, a correction must be made to adjust for the slower speed of sound waves in
silicone oil. Scleral buckling surgery often leads to an elongated eye, but this results in a measured increase the axial length.
Prior keratorefactive surgery may lead to small changes in axial length, but most of the error inherent in these A-scans is
related to difficulty in achieving accurate keratometric readings.

Question 124 of 130


For what condition is "in-the-bag" posterior chamber intraocular lens implantation contraindicated?

Angle-closure glaucoma

Fuchs heterochromic uveitis

Large posterior capsular tears

Zonular weakness
Please select an answer
Feedback: "In-the-bag" posterior chamber intraocular lens implantation is the preferred location for eyes following cataract
surgery. However, eyes with posterior capsular rupture should generally have sulcus IOL placement, or ACIOL placement in
the absence of anterior capsular support. Many eyes with locailzed zonular weakness may still have "in the bag" IOL
placement if the area of zonular weakness is small and localized. Capsular tension rings may facilitate "in the bag"
placement of IOLs in some cases with larger areas of capsular weakness. Fuch's and angle closure glaucoma are generally
not affected by IOL location, but in the case of angle closure, peripheral iridectomies may be useful if a patent iridectomy is
not already present.

Question 125 of 130


When analyzing the refractive error following cataract surgery, what effect would a 1.00 D error in keratometry measurement
have on the post-operative correction?

1.00 D

0.50 D

2.00 D

No error
Please select an answer
Feedback: Based on a commonly used regression formula, P = A - 2.5L - 0.9K (where P = IOL power, A = A-constant, L =
axial length, and K = kertatometry), an error in keratometry will lead to an almost 1:1 error in IOL power.
Question 126 of 130
According to the the Endophthalmitis Vitrectomy Study (EVS), what is the preferred treatment for acute post-cataract
endophthalmitis in an eye with LP vision?

Vitrectomy followed by periocular injection of steroid

Vitrectomy followed by intraocular injection of antibiotics

Vitreous tap/biopsy followed by q1 hour topical 4th-generation fluoroquinolones

Vitreous tap/biopsy followed by intraocular injection of antibiotics


Please select an answer
Feedback: The EVS determined that, if vision is hand-motion or better, vitreous tap/biopsy should be performed
and followed by intraocular injection of antibiotics. If vision is worse or equal to light-perception, immediate vitrectomy should
be performed (3-port for biopsy/cultures and sensitivities) followed by intraocular injection of antibiotics. Within the EVS, the
threshhold for hand motions vision was a measurement at three feet or more. If hand motions was present at less than
three feet, vision was recorded as light perception.

Question 127 of 130


A patient with intravitreal silicone oil requires cataract extraction. Which one of the following intraocular lens (IOL) types is
the best implant choice?

Acrylic

Biconvex

Square-edged

Toric
Please select an answer
Feedback: Wetting is the affinity of a liquid to a solid surface. When silicone oil contacts silicone IOLs, the IOL becomes
coated with oil. The oil surface induces irregular astigmatism and cannot be removed in a physiologic environment. Acrylic
IOLs do not have an affinity for silicone oil and is an ideal material to avoid silicone oil adherence. In an eye with silicone oil
in the vitreous cavity, silicone and biconvex IOLs should be avoided. Toric IOLs are preferred for corneal astigmatism
correction, and square-edged optics are most useful for preventing posterior capsule opacity.

Question 128 of 130


When considering optimizing cataract surgery outcome in a uveitis patient, what is the minimum duration that inflammation
be controlled prior to surgery?

1 week

2 months

1 month

3 months
Please select an answer
Feedback: Cataract surgery can be an inciting factor in stimulating a severe inflammatory reaction in a patient with uveitis.
The more recent the active inflammation, the greater the risk of a flare. Even with a prolonged quiescent period before
surgery, prophylactic anti-inflammatory medication should be used preoperatively. The minimum preferred quiescent period
of uveitis control is 3 months. Lesser intervals increase the risk of uveitis reactivation.
Question 129 of 130
What is the most likely cause of an unexpected postoperative refractive error following cataract surgery?

Selecting an inaccurate intraocular lens (IOL) power calculation formula

Inaccurate axial length measurement

Inaccurate keratometry

Poor surgical technique


Please select an answer
Feedback:

The SRK regression formula [P = A - (2.5L) - 0.9K] shows the importance of axial length in IOL power determination. Very
small errors in axial length measurement can lead to large errors in IOL power. Although kertometric errors can occur
(especially in patients with previous refractive surgery), in routine cases a small error in axial length measurement is more
likely.

Question 130 of 130


What antibiotic prophylaxis has been shown to decrease the incidence of endophthalmitis following cataract surgery?

Topical antibiotics on the day of surgery

Intracameral cephalosporins

Topical antibiotics three days before surgery

Postoperative topical antibiotics


Please select an answer
Feedback: The ESCRS randomized, prospective trial of endophthalmitis demonstrated the efficacy of intracameral
cefuroxime in preventing endophthalmitis. Although commonly used, the pre-, post-, and perioperative application of topical
antibiotics have not been shown to be effective in preventing endophthalmitis in a randomized prospective study.

Collected from www.aao.org (self assessment)


By Dr. AlBaraa AlQassimi

Vous aimerez peut-être aussi